Download Document 4487866

Document related concepts

Compartmental models in epidemiology wikipedia , lookup

Child Protective Services wikipedia , lookup

Child protection wikipedia , lookup

Transnational child protection wikipedia , lookup

Child migration wikipedia , lookup

Infection wikipedia , lookup

Syndemic wikipedia , lookup

Eradication of infectious diseases wikipedia , lookup

Unaccompanied minor wikipedia , lookup

Marburg virus disease wikipedia , lookup

Pandemic wikipedia , lookup

Canine distemper wikipedia , lookup

Infection control wikipedia , lookup

List of medical mnemonics wikipedia , lookup

Transcript
Crocs
Scarlet fever
A 7 y.o. girl fell ill abruptly: fever, headache, severe sore throat, vomiting. Minute bright red rash
appear in her reddened skin in 3 hours. It is more intensive in axillae and groin. Mucous membrane of
oropharynx is hyperaemic. Greyish patches is on the tonsils. Submaxillary lymph nodes are enlarged
and painful. What is your diagnosis?
*Scarlet fever
Measles
Rubella
Pseudotuberculosis
Enteroviral infection
Patient R., 9 y., complains on the appearance of the erythematous precipitations, which began from the
retroauricular sections, for the elongation of 72 hours they were extended to entire skin. Rashes
preceded an increase in the temperature of body to 39,5°C, general weakness, cough of bronchial
nature. It is known from anamnesis that 15 days ago the classmate was hospitalized into the infectious
hospital. What is the diagnosis?
*Scarlet fever.
Measles.
The chickenpox.
Pseudotuberculosis.
The German measles.
A 8 - year-old boy full ill abruptly fever up to 39 C, headache, recurrent vomiting, sore throat. There
was punctiform rash on reddened skin on his trunk, upper part of the abdomen, and in the skin folds,
especially in axillae, cubital, inguinal. The soft palate is covered with erithematous punctiform lesions
and scattered petechiae. Tonsils were enlarged, reddened with superficial necrosis in the form of
patches. Submaxillary lymph nodes were enlarged, dense and tender. BP is 140/90; Ps is 120 per min.
What is your diagnosis?
Adenoviral
infection *Scarlet
fever Diphtheria
Measles
Leukosis
A 8- year-old girl complained of fever up to 38,6 C, sore throat, rash. In medical examination lacunar
tonsillitis, hyperemia and enanthema of soft palate, punctiform rash which localized mainly on flexor
surface of the extremities was discovered. Nasolabial area was pale. Which antibacterial medications
should be administered?
Gentamycin
*Penicillin
Chloramphenicol
Lincomycin
Tetracycline
A 3-year-old child has a typical mild form of scarlet fewer and is treated at home. Which antibacterial
medication should be administered for etiotropic therapy?
*Erytrimycin
Gentamycin
Chloramphenicol
Doxycycline
Tetracycline
1
A 8-year-old boy has scarlet fewer and takes antipyretic and vitamins. Weakness, anorexia, skin
pallor, face edema, oliguria occurs on the 14 th day of illness. Specific gravity of urine is 1030, protein
level is 1 g/1, erythrocytes are 60 in the field vision. Hyaline cylinders are present too. What is
complication has occurred?
Tumour of urinary
bladder Pyelonephritis
Urolhhic disease
*Glomerulonephritis
Polyps of urethra
The patient of 14 years old in 2 weeks after a pharyngitis has started to complain of a ferves cence up
to 38,0°С, the general delicacy, dyspnea during walking, tumescence and pain of joints of flying
character. Objectively: cyanosis of lips, pulse - 100 impacts in a minute, weak filling, rhythmical. The
left border of heart is displaced outside from mediaclavicular lines on 1 sm. The first tone on an apex
weakened, gentle systolic hum is auscultated. What is the most probable etiological factor could cause
this syndrome?
Pneumococcal.
Virus.
Staphylococcus. *βhemolytic streptococcus.
Mushrooms.
The child of 5 years, did fall ill acutely, did increase the temperature of body to 38°C, complaint of the
sore throat when swallowing, there was a disposable vomiting, appeared punctuate exanthema against
the hyperemized background. A district pediatrician diagnosed it "Scarlet Fever", has appointed a
hospital at home. What medication should I choose for causal treatment?
*Penicillin.
Tetracycline.
Acyclovir. SulfatePolymyxin M.
Interferon.
The boy of 10 years old in 2 weeks after angina had edemas of the face and moderate back pain.
Objectively: a body temperature is 37,5°С, a BP - 100/80 mm hg. Urine: fresh erythrocytes are up to
100 in sight, protein - 2,20 gpl; hyaline cylinders are up to 10 in sight, relative density - 1, 020. What is
the most probable diagnosis?
Acute pyelonephritis.
*Acute glomerulonephritis.
Cancer of a kidney. Chronic
glomerulonephritis.
Urolithiasis.
A 10-year-old girl was admitted to a hospital with carditis presentations. It is known from the
anamnesis that two weeks ago she had exacerbation of chronic tonsillitis. What is the most likely
etiological factor in this case?
*Streptococcus
Staphylococcus
Pneumococcus
Klebsiella
Proteus
In 3 weeks after the child has had angina, hе still has weakness, flaccidity, subfebrile temperature,
2
enlarged mandibular lymph nodes. Tonsils are quaggy, densely soldered with handles; in lacunas we
can see purulent plugs. What is the most probable diagnosis?
Peritonsillitis.
Chronic pharyngitis.
Acute lacunar angina.
*Chronic tonsillitis.
Tumour of tonsils.
The girl of 12 years old has arrived in cardiologic department of the hospital with manifestations of a
carditis. Two weeks ago she had lacunar angina. What is the most possible etiological factor of a
carditis in this case?
Staphylococcus.
Klebsiella.
*Streptococcus.
pneumococcus.
Proteus.
In the child of 6 years 10 days after transferred angina on the skin of upper and lower extremities
appeared symmetrically located papular - hemorrhagic rash with sizes from 3 to 5 mm in the diameter;
swelling and pain in talocrural joints. In the clinical analysis of the blood: WBC. - 3,9*10*12 of/l, Hb 124 g/l, TsP. - 0,9, thrombocytes - 250*109/l, L. - 15,0*109/l, s. - 7, 3. - 8, sg. - 70, l. - 12, m. - 3, ESR
- 25 mm/h. What does be the basis of the development of hemorrhagic syndrome?
Reduce the number of platelets.
Reducing the concentration of plasma clotting
factors. *The defeat of the vascular wall.
No retraction of the blood-convolution.
Violation of platelet adhesion.
Girl with chronic tonsillitis after supercooling complains on the pain in the back. Urine of the color of
„ meat mud ", AD is increased to 150/100 mm Hg. In anamnesis allergic reaction to Ampicillin. What
antibiotic you will appoint for the purpose of the preventive maintenance of allergic reaction?
*Erythromycin.
Gentamicin.
Biseptol.
Bitsillin (benzathine penicillin
G). Ampicillin.
The child of 5 years, fell ill acutely. Doctor established diagnosis ―scarlet fever‖. Child attends
children's pre-school establishment. Indicate the period of the quarantine, which is superimposed on
the contact children in kindergarten.
*7 days.
10 Days.
14 Days.
21 Day.
24 Days.
Masha B. 8 years old suffering from scarlet fever. Was obtained the treatment: paracetamol, vitamins.
The apathy appeared on 14 days of disease, was reduced appetite, appeared pallor of the skin, pastiness
and edemas of face, oliguria. Urine: specific gravity 1030, protein 1 g / l, erythrocytes 60 in the field of
view, hyaline cylinders. What complications have developed in a girl? It.
Pyelonephritis.
Urolithiasis.
Tumor-bladder.
3
*Glomerulonephritis.
Polyps of the urethra.
To child of 5 years. 2 weeks it ago suffered angina. Complaints of mother on poor appetite, disrupted
sleep. With an objective research is revealed heart rate 100 in 1 minute. How this index should be
estimated?
*Age norm.
Tachycardia.
Bradycardia.
Respiratory arrhythmia.
Paroxysmal tachycardia.
Ten days ago 6 years old girl endured angina. To the period of appeal complains on the general
weakness, reduction in the appetite, two-fold vomiting, headache and pain in the back. It is objective:
the skins of pale pink color, the pastiness of face, the positive symptom of Pasternatskiy. Blood:
normochromic anemia, RES- of 15 mm/h, neutrophilic leukocytosis. In the analysis of the urine:
protein 1,5 g/l, the lixiviated erythrocytes to 20 into the field of sight, transparent cylinders 5 in the
field of sight. Your diagnosis:
*Acute glomerulonephritis.
Acute pyelonephritis.
Chronic pyelonephritis.
Chronic glomerulonephritis.
Subacute malignant glomerulonephritis.
In the child of 10 years against the background of the aggravation of chronic tonsillitis on 10th the day
the pain in the region of heart appeared. State of the child is of average gravity, the temperature of
37,8°C, FCR- of 93 beatings/min, weakening of the І tone above top, apical systolic noise. In the
blood: leukocytes to 12*109, SES- of 28 mm/hour, the level of antistreptolysin - O of serum - 450
UP/ml, SRP - (-). On ECG: lengthening atrioventricular conductivity, a change in the final part of
QRST. By the optimum version of the antibacterial therapy of myocarditis there is the designation:
*Benzylpenicillin.
Levomycetin.
Erythromycin.
Amoxycillin.
Amikacin.
In boy 14 years against the background of chronic tonsillitis, maxillary sinusitis appeared the sensation
of stoppages in the section of heart and additional pulse beatings. FCR- of 83 beatings/min. On ECG:
after every two sinus reductions the pulses, in which the absent tooth R, QRS has duration more than
0,11 s, acutely deformed, discordant T-wave, regularly appear, complete compensating pause after
which is recorded. Indicate the character of the disturbances of the rhythm:
*Extrasystolia according to the type of
trigeminy. Extrasystole according to the type of
bigeminy. Partial A- V blockade.
Complete A- V blockade.
Blockade of the left leg of His's beam.
4
Pseudotuberculosis
The child of 8 years, did fall ill acutely, temperature 38,5°C, vomiting, abdominal pain. With the
inspection for 3 days are revealed the yellowness of the skin and scalars, on the skin of body, brushes
and feet hyperemia, punctulate rash, increase in the liver, spleen. n the eve of the disease ate salad with
fresh cabbage. What is the most likely diagnosis?
*Pseudotuberculosis.
Viral Hepatitis.
Scarlet fever.
Enterovirus infection.
Infectious mononucleosis.
A 10-year-old girl complained of fever, rash, abdominal pain, artralgiae on admission. In medical
examination scarlet fever-like rash, symptoms of ,,hood", ,,gloves" and ,,socks" have been discovered.
What is your preliminary diagnosis?
Measeles
Allergic rash
Rubella
Scarlet fewer
*Pseudotuberculosis
A 5-year-old boy fell ill abruptly: fever up to 38,5, sore throat, abdominal pain. Scarlet fever-like rash,
artralgiae and frequently passed stools occurred on the 4l day f illness. In medical examination
hyperemia and edema of skin on the face, hands and feet, hyperemia of palatal arches, soft palate,
uvula back walls of pharynx, ,,raspberry tongue", pains in epigastria and near the umbilicus was
discovered. Watery stools occurred 4-6 times a day with small admixture of mucus. Padalka's
symptom was positive. What's your preliminary diagnosis?
Enteroviral infection
Scarlet fewer
Abdominal typhoid
*Pseudotuberculosis
Rubella
A 7-year-old boy has moderate form of pseudotuberculosis. Which antibacterial medication should be
administered for etiotropic therapy?
Penicillin
Ampicillin
*Chloramphenicol
Erytromycin
Cefalosporins of the first generation
A 7-year-old boy fell ill abruptly: fever up to 40 C with rigor, artralgia, myalgia, abdominal pain.
Polymorphic rash with macular and scarlet fever like elements appeared on the 4th day of illness.
Ictericity of skin and sclera occurred on the 6th day of illness. Liver and spleen were enlarged and
tender. Padalka's symptom was positive. What's your preliminary diagnosis?
Measles
Rubella
Scarlet fewer
*Pseudotuberculosis
Abdominal typhoid
5
Measles
Child of 5-years. Fifth day of illness. Objective: conscious, listless. Face is puffy. Catarrhal
conjunctivitis, scleritis. The skin behind the ears, on the face – there is a bright, red media papular
rash, sometimes the elements of coalesce. In the throat - diffuse hyperemia, on the soft palate enanthema. Is nott immunized. Your diagnosis?
*Measles.
Rubella.
Scarlet fever.
Pseudotuberculosis.
Allergic dermatitis.
Child of 2 years old, who has arrived into the hospital with pneumonia, has nevuses pigmentosus on a
skin. From anamnesis we have known that before pneumonia the child had fever and rash. What
disease had the child?
Rubella.
*Measles.
Scarlatina. ЕСНОexanthema.
Chicken pox.
There is a child of 1 year old. Against what disease there has come time to do planned
immunization? *Measles.
Tuberculosis.
Whooping cough.
Diphtheria.
Poliomyelitis.
A 5-year-old boy has been ill for 5 days. The disease had abrupt onset with fever up to 38,1 C, cough,
conjunctivitis, edema of eyelids. Maculopapular rash appears on his face, neck and behind his ears on
the 5th day of the disease. The next day the rash spreads to his trunk. What is your diagnosis?
Adenoviral
infection *Measles
Rubella
Infectious mononucleosis
Scarlet fever
A 3 year old child has been suffering from fever, cough, coryza, conjunctivitis for 4 days. He has been
taking sulfadimethoxin. Today it has fever up to 39oC and maculopapular rash on its face. Except of
rash the child's skin has no changes. What is your diagnosis?
Scarlet fever
Rubella
Pseudotuberculosis
*Measles
Allergic rash
A child, aged 4, has being ill for 5 days, suffers from cough, skin rash, t0- 38,20С, edema of the face,
photosensitivity, conjunctivitis. On the face, neck, upper part of the chest there is bright maculopapular
rash with areas of merging. Hyperemic throat. Seropurulent nasal discharge. In lungs there are dry
crackles. What is the most probable preliminary diagnosis?
Scarlet fever
Enterovirus exanthema
*Measles
Adenovirus infection
6
Rubella
A 7-year-old boy has been ill for 2 days. He complains of cough, coryza, headache, fever up to 38C.
There is conjunctival hyperemia, epiphora, enanthema on the mucosa of the soft and in part of the hard
palate, whitish papules looking like poppy-seed, surrounded by a narrow band of hyperemia opposite
the molar teeth. What is your diagnosis?
*Measles
Rubella
Scarlet fever
Enteroviral infection
Adenoviral infection
A child, aged 4, has being ill for 5 days, suffers from cough, skin rash, t° - 38,2°C, facial hydropy,
photosensitivity, conjunctivitis. On the face, neck, upper part of the chest there is bright maculopapular
rash with areas of merging. Hyperemic throat. Seropurulent nasal discharge. In lungs there are dry
crackles. What is the most probable preliminary diagnosis?
Adenovirus
infection. Rubella.
*Measles.
Scarlet fever.
Enterovirus exanthema.
A child is 4 years old, has been ill for 5 days. There are complaints of cough, skin rash, t-38,2ºC, face
puffiness, photophobia, conjunctivitis. Objectively: there is bright, maculo-papulous, in some areas
confluent rash on the face, neck, upper chest. The pharynx is hyperaemic. There are seropurulent
discharges from the nose. Auscultation revealed dry rales in lungs. What is the most likely diagnosis?
*Measles
Adenoviral
infection Scarlet
fever Rubella
Enterovirus exanthema
A 4-year-old child on the 5th day of illness complains of cough, rash on the skin. Temperature is
38,2°C, face is puffy, photophobia, conjunctivitis. There is a bright papulomacular rash on the face,
neck, upper half of the thorax. Pharynx is hyperemiated. There are serous and purulent discharge from
nose, dry rales in the lungs. What is your preliminary diagnosis?
*Measles.
German measles.
Enterovirus infection.
Adenovirus infection.
Scarlet fever.
A 2-year-old boy is not vaccinated against measles. He had been in touch with the child with catarrhal
period of measles and took immunoglobulin. He can fall ill within?
28 days
17 days
*21 days
14 days
7 days
A 9-year-old child is ill for 5 days. Physical examination: Conscious, inert. Puffy face. Catarrhal
conjunctivitis, scleritis. Bright-red papular middle-spotted skin rash on face and behind ears.
Somewhere elements merge. Diffuse hyperemia in pharynx. Soft palate shows enanthema. The child is
7
not vaccinated. What is the most likely
diagnosis? Allergic dermatitis.
Pseudotuberculosis.
*Measles.
Scarlet fever.
Rubella.
A 5-year-old child developed an acute disease starting from body temperature rise up to 38,5ºC,
running nose, cough and conjunctivitis. On the 4th day the child presented with maculo-papular rash
on face. Body temperature rose again up to 39,2ºC. Over the next few days the rash spread over the
whole body and extremities. Mucous membrane of palate was hyperaemic, there was whitish
deposition on cheek mucous membrane next to molars. What is your provisional diagnosis?
*Measles
Acute viral respiratory
infection Yesinia
Enterovirus
diseases Rubella
Child is ill the 5th day. Disturb the temperature of body increased to 39-40°C, persistent, frequent,
deep, unproductive cough, photophobia, obstruction of nose. It is objective - the puffiness of face, the
mucous membrane of mouth friable, hemorrhagic enanthema before the soft palate, remainders of the
spots of Filatov-Koplik’s. Your diagnosis?
Scarlet fever. German
measles. Adenovirus
infection.
*Measles.
Influenza.
To child was carried out 1 year. Against what illness did come the time to conduct planned
vaccination?
*Measles.
Tuberculosis.
Whooping cough.
Diphtheria.
Poliomyelitis.
In the child of 5 years district doctor diagnosed measles, the first day of rash. For what period it is
necessary to isolate child, if disease does flow without the complications?
*Up to 5 days the rash.
Up to 10-day eruption.
Before the advent of pigmentation.
Up to 21 days from the onset of the disease.
Up to 17 days from the onset of the disease.
The child of 6 years, fell ill with crust. became ill with measles. The family has another child, 4 years
old, who attends kindergarten. Concerning contact of a healthy child was put immunoglobulin. Name
the period of quarantine.
*21 day.
7 Days.
14 Days.
17 Days.
30 Days.
8
Rubella
A 2-year-old girl has been ill for 3 days. Today she has low-grade fever, severe catarrhal symptoms,
non-abundant maculopapular rash on her buttocks and enlarged occipital glands. What is your
diagnosis?
*Rubella.
Pseudotuberculosis.
Measles.
Scarlet fever.
Adenoviral infection.
The two years old child has fevers up to 37,2 °С and rash on skin. Objectively: general condition is
satisfactory. There is macular rash of pink color on skin of back and extension surface of extremities.
Mucous tunic of palatine arches is moderate hyperemic and has fine macular rash. There are
insignificant mucous discharge from a nose and enlargement of occipital lymph nodes. What is the
most probable diagnosis?
Enterovirus exanthema.
Scarlatina.
*Rubella.
Chicken pox.
Measles.
A 2-year-old girl has been ill for 3 days. Today she has low-grade fever, severe catarrhal symptoms,
non-abundant maculopapular rash on her buttocks and enlarged occipital glands. What is your
diagnosis?
Adenoviral
infection. Measles.
*Rubella.
Pseudotuberculosis.
Scarlet fever.
A 2 year old girl has been ill for 3 days. Today she has low grade fever, severe catarrhal presentations,
slight maculopapular rash on her buttocks and enlarged occipital lymph nodes. What is your
diagnosis?
*Rubella
Scarlet fever
Measles
Adenoviral infection
Pseudotuberculosis
A 5-year-old child has low-grade fever, enanthema, enlarged post-auricular cervical and suboccipital
lymph nodes, pale red small macular rash, which localizes mainly on the extensor surface of the
extremities, back and buttocks. What is your diagnosis?
Infectious mononucleosis
Measles
Scarlet fever
*Rubella Enteroviral
infection
A 2-year-old child full ill acutely: ever and rash appeared. Now it has slight malaise, coryza and
cough, enanthema on its soft palate. There is pale red small macular rash on its back and buttocks.
Post-auricular cervical and subboccipitall lymph nodes are the size of a large pea, hard and slightly
tender on palpation. What is your diagnosis?
9
*Rubella
Measles
Scarlet fever
Infections mononucleosis
Enteroviral infection
A baby was born on the 8 th month of gestation. It had microcephalia, cataract, cardiac defects. On the
2 nd month of the pregnancy its mother had low-grade fever, enlarged lymph nodes, and pale red small
macular rash on her face, trunk and extremities. These signs disappeared after 3 days without residual
occurrence. What is your diagnosis?
Toxoplasmosis
Cytomegaloviral infection
Herpetic infection
Chlamydic infection
*Congenital rubella
More common congenital rubella
syndrome Hydrocephaly
Vesicular rush
Interstitial
pneumonia Thyroid
disease *Hearing loss
10
Varicella
A child, aged 5, is ill with fever, vesicular rash mainly on the trunk and head skin. On the 8 th day there
appeared severe headache, ataxia, lethargy, movement discoordination, tremor of the extremities. On
the second wave of the fever encephalitis is diagnosed. Complication of what decease can be
encephalitis in this case?
Herpetic infection.
Enterovirus
infection. Measles.
*Chicken pox.
Rubella.
A 5-year-old child had strong headache, vomiting, ataxia, dormancy, discoordination of movements,
tremor of the extremities on the 8th day of the disease. It was followed by rise in body temperature,
vesicular rash mainly on the skin of the body and the hairy part of the head. At the second wave of the
fever a diagnosis of encephalitis was given. What disease complicated encephalitis in this case?
*Chicken pox
Measles German
measles
Enterovirus ifection
Herpetic infection
A 7 year old girl has mild form of varicella. Headache, weakness, vertigo, tremor of her limbs, ataxia,
then mental confusion appeared on the 5th day of illness. Meningeal signs are negative. Cerebrospinal
fluid examination is normal. How can you explain these signs?
*Encephalitis
Meningitis
Meningoencephalitis
Myelitis
Neurotoxic syndrome
On the 21 day after appearance of vesicular chickenpox rash a 7-year-old child developed ataxia,
nystagmus, intention tremor, muscle hypotonia. Liquor analysis shows a low-grade lymphocytic
pleocytosis, slightly increased protein rate. What complication is it?
*Encephalitis
Purulent meningitis
Pneumonitis
Acute nephritis
Postherpetic neuralgia
A 9-year-old boy has been ill for 2 days. Now he has fever up to 37,5°С, maculae, papules, vesicles on
his skin, skin of the head. Vesicles are round, situated on an unindurated base, surrounded by
erythematous corona. What is your diagnosis?
*Varicella
Smallpox
Scarletfewer
Herpetic infection
Streptodermia
A 7-year-old girl has been in contact with a patient with herpes zoster. Fever up to 39,3°С,
polymorphic rash (maculae, papules, crusts, vesicles) on her skin and mucous membranes of oral
cavity have occured on the 7th day of illness. What is your diagnosis?
*Varicella
11
Smallpox
Herpes simplex
Herpes zoster
Streptoderma
A 5-year-old child had strong headache, vomiting, ataxia, dormancy, discoordination of movements,
tremor of the extremities on the 8th day of the disease. It was followed by rise in body temperature,
vesicular rash mainly on the skin of the body and the hairy part of the head. At the second wave of the
fever a diagnosis of encephalitis was given. What disease complicated encephalitis in this case?
Measles. *Chicken
pox. Herpetic
infection.
Enterovirus ifection.
German measles.
The patient of 13 years old became ill acutely: T- 37,3°С, there is maculopapular, vesicular rash on a
trunk, extremities, scalp. For 6-th day the condition of the patient has worsened: T - 39°С, flaccidity,
headache, giddiness, unsteadiness of a gait, instability in Romberg's pose. What is your diagnosis?
*Chickenpox's encephalitis.
Herpetic encephalitis. Lyell's
disease.
Shingles.
Sepsis, hematosepsis.
A 4-year-old girl has severe form of varicella. What medication should be administered except
one: *Steroid hormones
Acyclovir
Specific immunoglobulin
Recombenated interferon Inductor
of endogenous interferon
A 8-year-old boy is not ill with varicella. He had been in touch with the child with varicella. He can
fall ill within.
28 days
17 days
*21 days
14 days
11-21 days
A 9 -year-old boy has fever, polymorphic rash - maculae, papules, vesicles and crusts on his face, skin
of the head, trunk, limb and extremities. Severe headache, vomiting, ataxia, slow down and
discoordination of motion have occurred on the 10th day of illness. What the complications is
diagnosed?
*Encephalitis Serous
meningitis
Neyrotoxycosis
Encephalitis reaction
Meningoencephalitis
A 6 -year-old girl has a lot of brown crusts on her face, trunk and extremities. What is the
illness? Caposi herpetiform eczema
Herpes simplex
Herpes zoster
12
*Varicella
Streptodermia
A 4 -year-old boy has fever, maculae, papules, vesicles and crusts on her face, skin of the head, trunk,
extremities and mucous membranes of oral cavity. He has been ill for 5 days. What the time will be the
patients isolated?
*In 5 days after of late rash
In all time of the disease
In 5 days begins of the
disease In 7 days after crusts
In 12 days after the onset of the disease
Mum with the child of 17 years old has apply to the hospital with complaints to the general weakness,
fever, morbid rash on skin of a trunk. The child is ill for 3 days. Objectively: there are grouped vesicles
with is serous-muddy and hemorrhagic contents on a background of hyperemia and edema, on a lateral
surface from the left side of a trunk. What is the most probable diagnosis?
*Herpes Zoster.
Simple contact Dermatitis.
Contact allergic Dermatitis.
Microbial eczema.
Herpetiform During's dermatosis.
13
Infectious mononucleosis
A 3-year-old girl has had fever, difficulty of nasal breathing, enlarged cervical lymph nodes.
Ampicillin was prescribed by a physician. Fever up to 38,3oC, lacunae tonsillitis, maculopapular rash
on the skin, enlarged liver and spleen had occurred on the 3rd day of illness. Blood count contained
leucocytosis, limphomonocytosis, atypical mononuclear cells - 20 %. What is your diagnosis?
Measles
Rubella
*Infectious mononucleosis
Scarlet fever
Adenoviral infection
A 10-year-old boy had complained of malaise, sore throat, difficulty of nasal breathing, fever up to 39
C on the 4th day of illness. Now he has pallor of the skin, edema of his upper eyelids, enlarged
postcervical lymphnodes. Mucous membrane of oropharynx is hyperemic, tonsils are enlarged,
covered by membranes, which are separated easily. Liver and spleen are enlarged. What is your
preliminary diagnosis?
Lymphogranulomatosis
Adenoviral infection
Tonsillar diphtheria
Scarlet fewer
*Infectious mononucleosis
A 2 year-old boy has been hospitalized with measles-like rash, tonsillitis, lymphoadenopathy,
difficulty of nasal breathing and hepatosplenomegaly. What is your preliminary diagnosis?
*Infectious mononucleosis
Cytomegaloviral infection
Measles
Serum sickness
Rubella
A 10-year-old child is seen in clinic and diagnosed an having infectious mononucleosis. Point to the
laboratory method which helps confirm the diagnosis.
Urinalysis
*Blood count with atypical mononuclear cells
Coagulogram
Smear from oropharynx
Transaminase level
A 3-year-old girl has had fever, difficulty of nasal breathing, enlarged cervical lymph nodes.
Ampicillin was prescribed by a physician. Fever up to 38,3 C, lacunae tonsillitis, maculopapular rash
on the skin, enlarged liver and spleen had occured on the 3rd day of illness. Blood count contained
leucocytosis, limphomonocytosis, atypical mononuclear cells-20 %. What is your diagnosis?
Measles
Rubella
*Infectious mononucleosis
Scarlet fever
Adenoviral infection
A 10-year-old boy had complained of malaise, sore throat, difficulty of nasal breathing, fever up to
39ºC on the 4th day of illness. Now he has pallor of the skin, edema of his upper eyelids, enlarged
postcervical lymph nodes. Mucous membrane of oropharynx is hyperemic, tonsils are enlarged,
covered by membranes, which are separated easily. Liver and spleen are enlarged. What is your
preliminary diagnosis?
14
Lymphogranulomatosis
Adenoviral infection
Tonsillar diphtheria
Scarlet fewer
*Infectious mononucleosis
A 2 year-old boy has been hospitalized with measles-like rash, tonsillitis, lymphoadenopathy,
difficulty of nasal breathing and hepatosplenomegaly. What is your preliminary diagnosis?
*Infectious mononusleosis
Cytomegaloviral infection
Measles
Serum sickness
Rubella
The child of 6 years fell ill with mononucleosis. What cells of the peripheral blood do confirm the
diagnosis of infectious mononucleosis?
*Mononuclears.
Lymphocytes.
Monocytes.
Neutrophils.
Basophils.
A 6 -year-old child has infectious mononucleosis. Which blood cells may confirm the
diagnosis? Polymorphonuclear cells
Basophilic leukocytes
*Atypical mononuclear cells
Eosinophils
Erythrocytes
Child of 5 years. Sick for day 5, the temperature of 38-39,5 ° C, sore throat when swallowing, pale,
lymphopolyadenopathia. Breath snoring with his mouth open. Throat is hyperemic, tonsils are
loosened with white-yellow membranes that proceeds from the lacunae. Liver is + 2.5 cm, spleen is +
2,0 cm In clinical analysis of blood there are leukocytosis, lymphocytosis, atypical mononuclear cells 25%. How research can confirm the diagnosis?
*Analysis of blood on the reaction of PaulBunel. Blood on sterility.
Blood on blood culture.
Blood on the paired sera.
Examination of blood by the thick film.
A 10-year-old child is seen in clinic and diagnosed an having infectious mononucleosis. Point to the
laboratory method which helps confirm the diagnosis.
Urinalysis
*Blood count with atypical mononuclear cells
Coagulogram
Smear from oropharynx
Transaminase level
15
Diphtheria
A 15-years-old girl complains of weakness, sore throat, fever up to 37,9 C. She has been will 3 days
after being in contact with the patient with tonsillitis. In medical examination congestive hyperemia
and moderate edema of oropharyngeal mucous membrane is found. Continuous grayish membrane is
the form of a film are located on the tonsils. The patches are taken off with difficulty and bleeding of
the mucous membrane is present. The patches spread on to the soft palate, the palatal arches and the
uvula. Regional lymph nodes are enlarged and painful. What is your diagnosis?
Lacunar tonsillitis
Follicular tonsillitis
*Tonsillar diphteria
Infections mononucleosis
Simanocesky-Vincent's disease
Child of 3 years, is ill 3-rd day. Temperature 38-38,5°C, insignificant pain with the ingestion, enlarged
anterior lymph nodes. With the inspection palatine almonds are swollen, the surface is covered with
the whitish-grey plaque with a smooth surface, closely associated with the nearby tissues. Is
established diagnosis ―diphtheria‖. What process is the basis of the formation of diphtheritic attacks?
Purulent inflammation.
Catarrh inflammation.
*Fibrinous inflammation.
Necrotic process.
Dystrophic process.
The sick child complains of the general weakness, giddiness, a fevers up to 37, 5°С, pharyngalgia,
edema of a neck, enlargement of submandibular lymph nodes. Objectively: mucous of guttur is
hydropic and cyanotic, tonsils are enlarged, covered by membranes which extend for their limits and
it's hard to take it out. What is basic mechanism of development of this disease?
Action of bacterial endotoxin.
Accumulation of suboxidated products.
*Action of bacterial exotoxin.
Allergic mechanism.
Bacteriemia.
A-5-year-old boy has been ill for 3 days: low-grade fever, malaise, anorexia, dry cough, then
hoarseness. Now he has striking pallor of the skin, perioral cyanosis, soundless cough, aphonia,
stenotic breath, retraction of complaisant places of the thorax, tachycardia 150 strokes per min. The
boy was not vaccinated. What is your preliminary diagnosis?
*Laryngeal
diphtheria Whooping
cough Foreign body
Epiglottidis
Parainfluenzal laryngotracheitis
Child, 3 years, the disease began from an increase in the temperature to 37,5°C, cough, it has been
hoarse voice. Not inoculated against diphtheria. On Day 3 of illness appeared short of breath, severe
symptoms of respiratory insufficiency, aphonic vote. Decongestants therapy is ineffective. The most
possible diagnosis?
Respiratory-large.
Foreign body airway.
Papilomatoz larynx.
*Diphtheria of the larynx.
Acute Allergic laryngitis.
16
At a child, sick with diphtheria after several days developed acute myocarditis. What pathogenetic
mechanism does answer for this complication?
*Influence of toxin.
The direct cytotoxic action of сorynebacteria.
Mean molecular peptide.
Products of the vital activity of
bacteria. Autoimmune defeat.
A 5-year-old child has tonsillar diphtheria, which is complicated by myocarditis. What is a
pathogenesis of this complication?
Autoimmune lesion
Cytotoxic influence of
corynebacteria *Diphtheritic toxin
Middlmolecular peptides
Metabolic product of bacteria
A 7-year-old child is sick for 2 weeks with running nose, was taking nasal drops. The boy suffers with
alimentary allergy. He applied to doctor due to suppurative and bloody discharges from nose,
maceration of nostrils and upper lip. Rhinoscopy results: there are whitish-grayish areas at nasal
septum. Mucous membrane of oropharynx is not changed. What is the most probable disease?
Allergic rhinitis.
Sinusitis (maxillar sinus)).
*Diphtheria of the nose.
Rhinovirus.
Adenovirus.
A 7 -year-old boy has been ill for 2 weeks, when coryza occured. Vaccination terms were not kept to.
Now he has difficulty of nasal breathing, sanioserous nasal discharge, excoriations on the skin at the
entrance into the nose and on the upper lip. Fibrinous membranes are discovered by rhinoscopy on the
mucous membrane of the nose. Mucous membrane of oropharynx is intact. What is your diagnosis?
*Nasal diphtheria
Adenoviral infection
Rhinoviral infection
Allergic rhinitis
Maxillary sinusitis
A 3-year-old boy has had severe form of tonsillar diphtheria. What complication may lead to lethal
outcome?
Nephrosonephritis
Pneumonia
*Myocarditis
Polyneuropathy Soft
palate paralysis
A 11-year-old girl had had tonsillitis and taken erythromycin at home 3 weeks ago. Now she
complains of nasal quality to the voice and nasal regurgitation .Soft palate is immovable in phonation.
Vaccination terms were not kept to. What is your preliminary diagnosis?
Poliomyelitis
*Diphtheritic
polyneuropathy Encephalitis
Botulism
Poliomyelitis-like form of enteroviral infection
17
The child of 6 years is ill with localized form of diphtheria of almonds. When it is possible to conduct
preventive inoculations?
After 1 month.
*At 6 months.
After 2 months.
After 1 year.
Vaccination not carried out.
In the child of 4 years, with the inspection are revealed pale hyperemia of almonds, their edema. On
both almonds the condensed stratifications of the gray-white color, with the smooth surface, which
protrude above mucous level, they are not removed by tampon. It was suspected diphtheria of the
stomatopharynx. Identify priority in treating a patient.
Appointment of
glucocorticoids. Antibacterial
therapy. Symptomatic therapy.
*Introduction antidiphtheritic
serum. Introduction Td toxoid.
In child at the 3rd day of disease - moderate sore throat, the subfebrile temperature of body. Dense dark
gray attack covers almonds and applies to front palatine arcs and uvula. Mucous membrane in the
place of defeat is hyperemized with the cyanotic nuance, is swelled. Preliminary diagnosis: extended
diphtheria of the stomatopharynx. By what preparation should be conducted specific detoxication?
Diphtheria toxoid.
Reopolyglukine.
Ringer's solution.
*Antidiphtheritic antitoxic serum.
5% Glucose.
18
Mumps
Child, 5 years old, acutely ill with fever up to 37,8 ° C, headache and swelling in the parotid region.
On day 2, appeared swelling in the left parotid region, tightly-elastic consistency, moderately painful
on palpation. What method of research is to confirm the disease at patient?
Smear-throat on flora. Blood
test for sterility. Blood test
for blood culture.
*Wipes from the nasopharynx to the virus.
Clinical analysis of blood-count with atypical mononuclear.
The child of four years old was examined by the local pediatrician. He complain to pain during a
mastication and opening of a mouth, headache and fevers up to 38,5°С. Objectively: in the region of
parotid salivary glands there are tumescence, moderately morbidity at a palpation, skin above it is not
changed. At the examination of a gutter we can see that aperture of Stenon duct is hyperemic. What is
the most probable diagnosis?
Sialadenitis. Diphtheria
of a guttur. Cervical
lymphadenitis.
*Epidemic parotitis.
Infectious mononucleosis.
A 4-year-old boy complains of fewer up to 38,9 C, pain in chewing and opening the mouth, headache.
Right parotis gland is enlarged and painful in palpation. The skin over the parotid glands does not
change. In lesion side Stenon's duct is reddened and edematous. What is your diagnosis?
Infectious mononucleosis
*Mumps
Rubella
Cervical lymphadenitis
Sialoadenitis
A 6-year-old boy complains of headache, malaise, pain in chewing, fever up to 39C.The following day
the enlarged parotid glands are noticeable. Swelled parotid glands obliterate the fosses are
retromaxillaris. The skin over the inflamed glands is tense and lustrous, remains of normal color.
Stenon's ducts are reddened and edematous. What is your diagnosis?
Tumor of parotid glands
*Mumps
Cervical lymphadenitis
Sialolithic disease
Toxic form of tonsillar diphtheria
A 7-year-old boy fell ill abruptly: fever up to 39C, headache, recurrent vomiting. Positive meningeal
signs are present. The boy had been in contact with patient with mumps. He is not vaccinated against
mumps. Which is more probability diagnosis?
*Parotitis meningitis
Meningococcal meningitis
Pneumococcal meningitis
Staphylococcal meningitis
Tuberculosis meningitis
A 10-year-old boy had mumps. On the 5th day of illness the child's general condition becomes worse.
Abdomen pain, nausea, anorexia and then vomiting occurred. Pain in the region of the left
hypochondria was determined by palpation. The amylase of the blood and diastase of the urine
19
increased highly. Which lesions
occurred? Appendicitis
*Pancreatitis
Mesadenitis
Cholecystitis
Gastritis
A 5-year-old boy had been ill 2 weeks ago. The disease began from dry cough, which increased
gradually. Paroxysmal cough appeared and whoops occurred. In paroxysm the patient's face is read,
the cervical wins are engorged. There are hemorrhages in the sclera. Leukocytes level is 16xl0 9/I,
lymphocytes-72%, ESR-4 mm/h. What is your preliminary diagnosis?
*Mumps Adenoviral
infection
Foreign body in respiratory tract
Parainfluenza with laryngotracheitis
Tuberculosis bronchodenitis
In a 10-year-old girl having mumps fever, recurrent vomiting and severe surrounding pain in upper
part of her abdomen occurred on the 4th day of illness. Which is laboratory test should be prescribed to
determine the diagnosis?
Bilirubin level
Glucose level
Thymol test
Activity of ALT
*Analyses of blood
In the family of two children, the elder fell ill to the parotitic infection of 5 days ago. Low-order of 5
years, f mumps was not sick, attends kindergarten hat are the GP on the contact of the child?
*Do not allow children to kindergarten from 11 to 21 days from the date of
contact. Vaccinate against mumps.
Assign antivirals. Enter
immunoglobulin.
Keep the garden from the moment of contact.
20
Whooping cough
Child is 1 month, suffering from whooping cough. In one of the fits of coughing the child turned blue
and stopped breathing. What causes apnea?
*Supraliminal inhibition of the respiratory
center. Mucous coat of the larynx.
Cerebral edema.
Laryngospasm.
Pulmonary edema.
The child of 5 years is ill for 2 weeks. First appeared the assaults of cough, then – reprise. Face of
patient reddens during the cough, the veins of neck will swell. The assaults of cough conclude with
vomiting. In the X-ray photograph - increased bronchial pattern. Blood tests: A - 16 g / L, lymph .72%, ESR - 4 mm per hour. What is the most likely diagnosis?
*Pertussis. Obstructive
bronchitis. Pneumonia.
Adenovirus infection.
Intraorbital foreign-body.
In child, sick with whooping cough, the basic clinical manifestation of disease is paroxysmal cough.
What pathogenetic mechanism does answer for this?
*Forming of center of excitation in CNS according to the type of whooping
dominant. Direct toxic action of agent.
Inflammation of the upper respiratory tract.
Formation of membranes in the upper respiratory
tract. Connection of secondary flora.
Two-year-old child coughs approximately one month. For a period of recent 3 weeks the cough
became attack-like. After the assault of cough - vomiting. It is objective: the puffiness of face. Subconjunctival hemorrhage to the right. On the skin of neck and of chest single petechiae; ulcer on the
bridle of language. Your diagnosis?
*Pertussis.
Tubercular
bronchoadenitis. Spastic
bronchitis. Hemorrhagic
vasculitis. RS- infection.
A 2-year-old girl complains of paroxysmal cough with whoops. There is edematous face, hemorrhages
in the sclera, shallow ulcer on the frenulum of the tongue. Auscultation reveals dry rales. Examination
of the oropharynx leads to cough paroxysms. What is your preliminary diagnosis?
*Whooping cough
Obstructive bronchitis
Foreign body in respiratory
tract Pneumonia
Stenosis laryngotracheitis
A 2 -year- old girl had been ill for 2 weeks. Now she has paroxysmal cough with whoop xnd vomiting,
hemorrhages in her sclera, shallow ulcer on the frenulum of the tongue. Auscultation reveals dry rales.
Point to medication for prophylaxis of this disease:
Antibiotics
*APDT-vaccine
ADT-anatoxin
21
ADT-M-anatoxine
Bacteriophages
A 1-year-old baby has whooping cough. In one of cough paroxysms the child becomes cyanotic and its
breathing disappears. Point to the cause of breathing disappearance.
*Inhibition of respiratory center
Edema of larynx mucous
membrane. Brain edema
Lung edema
Laryngospasm
In kindergarten the child of 5 years fell ill to the whooping cough. Name the period of the quarantine,
which is superimposed on the healthy children in the group.
7 Days
21 Day
*14 days
30 Days
45 Days
22
Polimyelities
A 1,5 y.o. child fell ill acutely with high temperature 38°C, headache, fatigue. The temperature
declined on the fifth day, muscular pain in the right leg occurred in the morning, there were no
movements and tendon reflexes, sensitivity was reserved. What is the initial diagnosis?
Viral encephalitis.
Hip joint arthritis.
*Polyomyelitis.
Polyartropathy.
Osteomyelitis.
2 years old child during 2 days had fever and insignificant catarrhal phenomena. On 4th day hi started
to pull the right foot, the temperature became normalized. The doctor has suspected a poliomyelitis.
What form of poliomyelitis is the most possible?
*Spinal.
Meningeal.
Abortive.
Bulbar.
Child 2 years old, desperately ill with a temperature increase up to 38,5° C, disposable vomiting,
diarrhea, slight cough, runny nose. After 3 days the temperature dropped to normal, and the child not
longer stand on his feet. The examination revealed limitation of movement, decreased muscle tone,
absence of tendinous reflexes in the lower extremities, more in the proximal segments. Sensitivity
saved. What is the most likely diagnosis?
Herpetic infection.
Meningococcal
infection. Influenza.
*Polio.
Rotavirus.
Child 5,5 years old fell ill acutely with increasing temperature up to 39,2° C, repeated vomiting,
coughing, runny nose. Disturbed sore throat, pain in arms and legs, in the spine, especially if he
wanted to sit down. In the throat is congestion. After 2 days - there is no active movement in the lower
extremities. Reduced tendon reflexes. Neck stiffness, Kernig symptom are positive. CSF: protein 0,33
g / L, cells - 65 cells in 1 ml of 65% of them - lymphocytes, 35% - neutrophils. Your diagnosis?
*Polio.
Viral encephalitis.
Enteroviral infections, serous
meningitis. Influenza.
Tuberculous meningitis.
The child of 12 months old had short febrile fever, weakness and diarrhea 8 days ago. Hi has flaccidity
of the right lower extremity now. The doctor suspects a poliomyelitis. What form of disease can it be?
*Spinal.
Meningeal.
Admixed.
Bulbar.
The child is ill with poliomyelitis. What term of regular medical check-up must be for the child after
recovery?
*For the moment of function
recovery. 3 month.
23
6 months.
1 year.
2 year.
A child is 9 months old. The patient's body temperature is 36,7ºC, the skin is pale, humid, there is pain
in leg muscles. There is no extremities mobility, sensitivity is present. The child has been diagnosed
with poliomyelitis. The causative agent of this disease relates to the following family:
*Picornavirus
Paramyxovirus
Tohovirus
Adenovirus
Rotavirus
A 1,5-year-old child fell ill acutely with high temperature 38ºC, headache, fatigue. The temperature
declined on the fifth day, muscular pain in the right leg occurred in the morning, there were no
movements and tendon reflexes, sensitivity was reserved. What is the initial diagnosis?
*Polyomyelitis
Viral encephilitis
Polyartropathy
Osteomyelitis
Hip joint arthritis
A 3-year-old child has been taken to a pediatrician. He has no recent history of any diseases. Objective
examination revealed no pathology of the internal organs. The child needs the routine immunization
against the following disease:
*Poliomyelitis
Diphtheria and tetanus
Measles, rubella, parotitis
Pertussis
Type B hepatitis
A 8-month-old not vaccinated baby fell ill acutely: fever up to 38,5ºC, anorexia, weakness, motor
anxiety, mild catarrhal signs and diarrhea. Loss muscle tone, absence of active movement and tendon
reflexes occurred on the right leg on the 5th day of illness. Cutaneous sensitivity is present. What is
your preliminary diagnosis?
*Polimyelities
Viral encephalitis
Acute respiratory viral
infection Acute intestinal
infection Osteomyelitis
A 2-year-old boy is having low-grade fever, mild catarrhal signs for 3 days. The body temperature
decreases on the 4th day of illness and flabby paralysis on his right leg occurs. What clinical form of
poliomyelitis is present?
*Spinal
Abortive
Meningeal
Pontine
Bulbar
A 18-month-old not vaccinated baby was hospitalized due to flabby paralysis of the left leg. There is a
loss muscle tone, absence of active movement and tendon reflexes. Cutaneous sensitivity is present.
Where is the CNS lesion localized?
24
Truncus cerebri
Posterior horns of spinal cord
*Anterior horns of spinal
cord Cerebellum
Pia matter
A child is 2 years old. The disease started sharply with rise of the temperature up to 38.8°C, vomiting,
headache, flaccidity, pains in the abdomen, softening of stool. In 3 days there were weakness in the
left leg, hypotonia, hyporeflexia, hypodynamia at the same place, the skin on the left leg was pale and
cold. What are the ways of distribution of the causative agent of this disease?
Respiratory
*Fecal - oral
Respiratory and fecal-oral
Respiratory and inoculable
Fecal - oral and parenteral
A child of 8 years old is hospitalized with complaints on sharp onset of the disease, febrile fever,
marked headache, repeated vomiting. On the 2-nd day of the disease the child developed nystagmus,
dysphagia and aphasia. Respiration is superficial, cough is silent. What form of poliomyelitis should
we think of?
*Nonparalytic, a meningeal
form Paralytic, a spinal form
Paralytic, a bulbar form
Nonapparent form
Paralytic, an encephalic form
A child of 4 years old is hospitalized with complaints on sharply developed paralysis of the right leg
against the background of temperature elevation up to 39°C. 4 days before there was a subfebrile fever
with insignificant catarrhal symptoms, pain in the abdomen, sweating. Poliomyelitis is suspected.
What symptoms, except one, will be characteristic of
poliomyelitis? Atonia (hypotonia)
*Adynamia (hypodynamia)
Areflexia (hyporeflexia)
Anesthesia (hyposthesia)
Atrophy (hypotrophy)
Child is ill with poliomyelitis. On what period is established clinic observation of it after the previous
disease?
3 months.
6 months.
1 years.
*Until of the restoration of the lost
functions. 2 years.
25
Meningococcal infection
Meningitis
A 5-year-old boy fell ill abruptly: fever up to 39,5ºC, severe headache, vomiting. Meningeal signs are
positive. Neutrophile pleocytosis are present in cerebrospinal fluid. Purulent meningitis is diagnosed.
What is the most probable etiology of meningitis?
*Meningococcus
Pneumococcus
Staphylococcus Hemophilus
influenza Micobacterium
tuberculosis
A 4-month-old-baby fell ill acutely: fever up to 39,5 C, vomiting, weakness. In 5 hours maculopapular
rash occurred on the lower extremities, the buttocks, the abdomen. After 10* hours from onset of the
disease hemorrhages rash then necroses appeared on the same parts of the skin. What is your
diagnosis?
*Meningococcal
infection Rubella
Measles
Influenza Hemorrhagic
vasculitis
A 5 year-old child fell ill sharply. Fever, intensive headache, repeated vomiting and meningeal signs
dominated in the clinical picture. In 2 days he died. On autopsy: soft brain membranes were sharply
sanguineous, impregnated with thick yellow-greenish exudates on the convex and basal surfaces of the
brain. The brain was swollen.
What disease are these changes characteristic
of? *Meningococcal infection
Herpetic infection,
Whooping-cough
Enteroviral
infection Measles
The child of 4 months old became ill acutely: temperature 38,5°С. Unitary vomiting, flaccidity. In 10
hours there was rash on breech and bottom extremities in the form of petechias, maculae and papules.
Some hemorrhagic elements are with a necrosis in the center. What disease can be the most probable?
*Meningococcemia.
Rubella.
Scarlatina.
Hemorrhagic
vasculitis. Flu.
A 4 month old child fell seriously ill: body temperature rose up to 38,5ºC, the child became inert and
had a single vomiting. 10 hours later there appeared rash over the buttocks and lower limbs in form of
petechiae, spots and papules. Some hemorrhagic elements have necrosis in the centre. What is the most
probable disease?
*Meningococcemia
Rubella
Influenza
Hemorrhagic
vasculitis Scarlet fever
26
Child, 3 years, fell ill with the night: the temperature of the body of 40°C, repeated vomiting,
headache. On the skin of body, extremities and face it appeared large sizes the hemorrhagic rash of
irregular star like shape, illegible meningeal symptoms. Your diagnosis?
Tuberculosis.
*Meningococcemia.
Enteroviral infection.
Staphylococcal
sepsis. Measles.
The child for 4 months old became ill acutely: temperature 38,5°С. Unitary vomiting, flaccidity. In 10
hours there was rash on breech and bottom extremities in the form of petechias, maculae and papules.
Some hemorrhagic elements were with a necrosis in the center. What disease can be the most
probable?
*Meningococcemia.
Rubella.
Scarlatina.
Hemorrhagic
vasculitis. Flu.
Child of 6 months, is taken to the hospital in critical condition. Acutely has being ill ill last night with
raising the temperature to 40 ° C, repeated vomiting, in 6 hours - appeared rash. Objectively, the 2 nd
day of illness: the condition was extremely serious, dramatically listless, There is general cyanosis of
the skin and mucous membranes, on the trunk, distal extremities abundant bluish color are stellar
eruption from 0,5 to 1,5 cm in diameter. Muscle stiffness in the neck, tension and pulsation large
fontanel are moderately expressed. Anuria. Your diagnosis?
*Meningococcal infection.
Staphylococcal sepsis.
Hemorrhagic vasculitis.
Flu, toxic form of haemorrhagic
syndrome. Thrombocytopenic purpura.
The child of 4 years old became ill acutely. At examination: the patient is excited, complaints to a
headache, double vomiting, body temperature is 39,2°С and there are individual petechias on the
thighs and crus and expressed muscle tension of a nape, consciousness is kept, no local signs. What
form of meningococcal infection has this child?
*Meningitis with
meningococcemia. Meningitis.
Meningococcemia.
Meningocephalitis.
Nasopharyngitis.
The child of 5 years old has arrived into the hospital with purulent meningitis that was caused by
meningococus. What kind of changes in liquor you are expecting?
*Neutrophilic pleocytosis.
Lymphocytic pleocytosis.
Normocytosis.
Hemorrhagic liquor.
Lower concentration of chlorides.
A 3 year old child fell acutely ill, body temperature rose up to 39,5oC, the child became inert, there
appeared recurrent vomiting, headache. Examination revealed positive meningeal symptoms, after this
lumbal puncture was performed . Spinal fluid is turbid, runs out under pressure, protein concentration
27
is 1,8 g/l; Pandy reaction is +++, sugar concentration is 2,2 millimole/l, chloride concentration - 123
millimole/l, cytosis is 2,35x109 (80% of neutrophils, 20% of lymphocytes). What is the most probable
diagnosis?
Serous tuberculous
meningitis Brain tumour
Serous viral meningitis
*Purulent meningitis
Subarachnoid haemorrhage
1n a 10-year-old baby septic form of meningococcal infection with infective toxic shock of the 1st
stage was diagnosed. Which medications should be given for start therapy?
Prednisolone and immunoglobuline
Prednisolone and penicillin
*Levomycetin sodium succinate and
prednisolone Penicillin and immunoglobulin
Immunoglobulin
A 1,5 y.o. child fell seriously ill: chill, body temperature rise up to 40,10С, then rapid dropping to
36,20С, skin is covered with voluminous hemorrhagic rash and purple cyanotic spots. Extremities are
cold, face features are sharpened. Diagnosis: meningococcosis, fulminant form, infection-toxic shock.
What antibiotic must be used at the pre-admission stage?
*Soluble Levomycetine succinate
Penicillin
Sulfamonometoxin
Lincomycin
Gentamycin
A 3 year old boy fell ill abruptly: fever up to 39ºC, weakness, vomiting. Hemorrhagic rash of various
size appeared on his lower limbs within 5 hours. Meningococcemia with infective - toxic shock of the
1 degree was diagnosed. What medications should be administered?
*Chloramphenicol succinate and prednisone
Penicillin and prednisone
Penicillin and immunoglobulin
Chloramphenicol succinate and interferon
Ampicillin and immunoglobulin
A child with meningococcus meningitis has been taking penicillin for 7 days. During last 4 days a
body temperature is normal. Meningitic signs are absent. When is it possible to cancel antibiotic?
If liquor cytosis is 100 or less, prevalence of neutrophils
If leucocytosis and left neutrophil shift are absent
*If liquor cytosis is 100 or less, prevalence of
lymphocytes If liquor cytosis is 150, prevalence of
lymphocytes Immediately
In a family, which consists of 4 persons from 3 till 30 years a cases of meningococcal infection occurs.
What urgent prophylaxis should be given among contacted persons?
*Antibiotics
Vaccination of antimeningococcal
vaccine Prophylaxis not conducted
Gargling of oropharynx by
antiseptics Bacteriophages
A 10-year-old boy fell ill abruptly after staying at a beach. Recurrent vomiting, severe headache, fever
28
appeared in the evening. Meningeal signs are positive. Cerebrospinal fluid is transparent, its pressure
increases, the number of CSF leukocytes is 350 cells /mm3 (lymphocytes -80%), glucose level-2.21
mmol/1, protein level-0.66 g/1. Pandi's reaction (+).What is your preliminary diagnosis?
Forest-spring encephalitis
Purulent meningitis
Tuberculous meningitis
Food poisoning
*Serous meningitis of viral etiology
A child is 2 years old. The disease began with rise of the temperature up to 39°C, repeated vomiting, in
some hours hemorrhagic rash developed on the skin of buttocks, shins, there were spasms. On
admission the condition was comatose, pulse was threadlike, 150 beats/min, arterial pressure 60/10mm
Hg., plentiful confluent hemorrhagic rash on the skin of buttocks, legs, abdomen, back.
What syndrome has developed in the child?
Acute cardiovascular insufficiency
Acute adrenal
insufficiency Acute renal
failure *Edema of the brain
Disseminated intravascular coagulation of blood (DIC) syndrome
The child of 5 years old was ill during 1 week. The body temperature was 37, 6°С, a tumescence in the
region of neck appeared. It was diagnosed: ARI, cervical lymphadenitis. Treatment: the erythromycin,
hot compress on a neck. On the background of treatment the body temperature has raised up to 39,
0°С, there was headache, repeated vomiting, meningeal syndrome. What researches should be done for
definition of the final diagnosis?
Puncture of a cervical lymph node.
General analysis of blood.
Investigation of sputum for secondary
flora. X-ray imaging of lungs.
*Spinal puncture.
Girl of 8 years. She complains of the appearance of skin rash in a child as spots of red, up to 5 mm.
The elements are arranged symmetrically, mainly in the elbow. What method can distinguish the
hemorrhagic spot of vascular?
*Clicking.
Percussion.
Grinding.
Examining.
Puncture.
During the inspection of the child of 1,5 years old it is established that it did fall ill acutely, it did
increase temperature to 39C, appeared vomiting to 5 times. Study of the nervous system showed
positive symptoms Kernig, Brudzinskogo. These symptoms relate to:
*Meningeal signs.
Syndrome discoordination.
Syndrome of motor disorders.
Reflex of the newborn.
No right answer.
Child 5 years old, fell ill suddenly: a high temperature up to 39,7° C, on the thighs and buttocks
appeared hemorrhagic rash star-shaped size from 0.5 to 3 cm, acrocyanosis, cold extremities, thready
pulse. Doctor ambulance suspected meningococceamia. What antibiotic must be appointed to child in
the prehospital stage?
29
Oxacillin.
Gentamicin.
*Levomitsetina
succinate. Cefazolin.
Rifampicin.
Children 6 years old fell ill with acute: fever up to 39,5 °C, appeared severe headache, nausea,
vomiting. On examination, the child is lethargic. There are expressed rigidity of occipital muscles,
positive Kernig - Brudzinskogo signs. Liquor is muddy, cells 1450 cells / mm (90% neutrophils),
protein - 1.8 g / l, reaction Pandy + + +. Which antibiotic should be appointed to start therapy?
*Penicillin.
Ofloxacin.
Tetracycline.
Erythromycin.
Rifampin.
Child 1 year old, is delivered into the hospital in the agonal state. It is ill the first day. Against the
background temperature 40C on the skin of lower extremities there is thick hemorrhagic-necrotically
rash, AD - 20/0 of mm mercury column BH - 44/min, pulse filamentary 200/min. Meningeal
symptoms are negative. After 1 hour the child died, in spite of taking resuscitative measures. Is
established the diagnosis of meningococcemia. Name the most possible reason for death.
*Hemorrhage in the adrenal.
Swelling of the brain.
Acute renal failure. Acute
heart failure. Acute
respiratory failure.
A child 7 years old, fell ill suddenly. Fever to 39°C, headache, repeated vomiting. On examination, the
patient are positive meningeal symptoms (muscle stiffness in the neck, Kernig symptom). Child 2
weeks ago contacted patients mumps infection in the classroom. Not immunized against mumps. What
is the most likely diagnosis?
*Mumps meningitis.
Enteroviral meningitis.
Meningococcal meningitis.
Pneumococcal meningitis.
Tuberculous meningitis.
In kindergarten to the meningococcus infection the governess fell ill, it was hospitalized. What
counterepidemic measure must be taken in focus?
Take the pin from the nasal wash for
viruses. Conduct an active immunization.
Assign antivirals.
*Get contact from nasopharyngeal swab for
meningococcal. Assign-antihistamines.
A 3-year-old boy suddenly fell ill with fever up to 39°C, weakness, vomiting. Hemorrhagic rash of
various size appeared on his lower extremities in 5 hours. Meningococcemia with infective-toxic shock
of the 1st degree was diagnosed. What medications should be prescribed?
Chloramphenicol succinate and interferon.
Penicillin and immunoglobulin.
*Chloramphenicol succinate and prednisone.
Penicillin and prednisone.
30
Ampicillin and immunoglobulin.
A 3-year-old boy fell ill abruptly: fever up to 39°C, weakness, vomiting. Hemorrhagic rash of various
sizes appears on his lower limbs in 5 hours. Meningococcemia with infective – toxic shock of 1 degree
was diagnosed. What medications should be administered?
*Chloramphenicol succinate and
prednisone. Penicillin and immunoglobulin.
Chloramphenicol succinate and interferon.
Penicillin and prednisone.
Ampicillin and immunoglobulin.
The child of 3th years old became ill acutely, the body temperature build-up to 39,5°С, child was
flaccid, had headache and repeated vomiting. At examination positive meningeal signs are appear. In
the results of lumbar puncture we can see that cerebrospinal fluid is muddy, flow out under greater
pressure, the contents of protein - 1,8 g/l; reaction Pandi +++, Saccharum - 2,2 millimole per liters,
chlorides - 123 millimole per liters, cytosis - 2350х10/6 (80 % of neutrophils, 20 % of lymphocytes).
What is the most probable
diagnosis? Serous meningitis.
Tumour of a brain.
Subarachnoidal hemorrhage.
*Purulent meningitis. Serous
tubercular meningitis.
A 5-year-old boy fell ill abruptly: fever up to 39,8°C, recurrent vomiting, severe headache.
Convulsions occur in 3 hours. Physician found out positive meningeal sign. Pleocytosis of 2500 cells
chiefly polymorphonuclear cells, elevated protein concentration and normal glucose concentration was
found in cerebrospinal fluid examination. What is your diagnosis?
Tuberculous meningitis.
Serous meningitis.
*Purulent meningitis.
Subarachnoidal
hemorrhage. Encephalitis.
A 3-year-old boy fell suddenly ill: fever up to 39°C, weakness, vomiting. Hemorrhagic rash of various
sizes appears on his lower limbs in 5 hours. Meningococcemia with infective - toxic shock of 1 degree
was diagnosed. What medications should be administered?
Penicillin and immunoglobulin.
*Chloramphenicol succinate and
prednisone. Chloramphenicol succinate and
interferon. Ampicillin and immunoglobulin.
Penicillin and prednisone.
A child of 3 years old fell ill 12 hours ago, the temperature is 39.6°C, with a headache and repeated
vomiting. Objectively: languid, sleepy, hemorrhagic stellate rash on the skin of buttocks and thighs.
The arterial pressure is 80/50 mm. Hg, symptoms of Kernig, Brudzinsky are expressed, there is rigidity
of the occipital muscles. In liquor: cytosis 5800 cells/mm3 (80 % - neutrophils), protein 1.8 g/l.
What preparation should be
administered? Erythromycin
*Levomycetin succinate
Penicillin
Gentamycin
Ceftriaxon
31
A 6-year-old boy fell ill abruptly: fever up to 39,5 C, severe headache, nausea and vomiting.
Meningeal signs are positive. Cerebrospinal fluid is turbid, cytosis is 1450/cells/mm3
polimorphonuclear cells-90%), protein level is 1.8 g/1, Pandi's reaction (+++) .What antibiotics should
be administered for the start therapy?
Tetracycline
Ofloxacin
*Cephalosporin
Rifampicin
Erytromicin
Drug of choice at the time of treatment of ІІ stages of the DIC-syndrome?
*Fresh frozen plasma.
Prednisolonum.
Heparin.
Cryoprecipitate.
Kontrical.
32
Dysentery
Child 1 of year fell ill suddenly: increased the temperature of body to 39,9ºC, appeared uneasiness,
vomiting, paroxysmal abdominal pain, frequent defecations up to 20 times in a 24 hour period.
Defecations are liquid; contain a large quantity of mucus and the admixture of the blood. The sigmoid
colon is condensed, unhealthy during the palpation. Put a preliminary diagnosis.
Cholera.
Staphylococcal gastroenterocolitis.
*Dysentery.
Salmonella.
Rotavirus gastroenteritis.
A child of 1 year old was admitted to the clinic with a severe form of intestinal dysfunction, signs of
neurotoxicosis. On bacterial examination Shigella of Grigoriev-Shiga. What is not characteristic of
Grigoriev-Shig’s dysentery?
Rapid development of colitis
syndrome *Mild course
Severe course
Hyperthermia
The expressed syndrome of infectious toxicosis
A child 3 years old is ill with a severe form of Flexner’s dysentery. What factor does not determine
morphological changes in an organism and severity of dysentery?
*Bacteriemia
Invasive features of the causative
agent An infecting dose
Endo-or exotoxin formation of the causative
agent Immunologic resistance of an organism
A child of 5 years old was admitted to the clinic with complaints on watery stool with mucus 5 times a
day, cramp-like pains in the abdomen. On bacteriological examination Shigella Sonnei was isolated.
The symptoms reflecting severity of the disease are:
Character of excrements
Frequency of excrements
Pains in the abdomen
Incompletely closed anus
*All answers are true
A-5-year-old boy fell ill acutely: fever up to 38,5C, anorexia, nausea, diarrhea, stools occur 8 times
daily, contain a lot of mucus and blood, thenesmus are present, sigmoid colon is tenderness and
hardened. What is your diagnosis?
Staphylococcal
enterocolitis Salmonellosis
*Dysentery
Escherhiosis
Viral diarrhea
A baby of 6 months is ill with a mild form of acute dysfunction of the intestine. On bacteriological
examination Shigella Sonnei was isolated. What morphological changes in the intestine prevail in a
similar case?
*Catarrhal colitis
Follicular colitis
Ulcerative colitis
33
Fibrinous colitis
Diphtheric colitis
The 7 years old child complains of the general weakness, spastic pain in the inferior part of a belly,
mainly in the left inguinal region, frequent fluid excrements up to 18 times a day with an admixture of
mucilage and blood. The beginning of disease was acutely, three days ago with a cold fit, fever and
headache. The general state is medium degree of gravity; temperature is 37, 8°С. Sigmoid intestine is
spastic and morbid. What is the most probable diagnosis?
Amebiasis.
Nonspecific ulcerative
colitis. Yersiniosis.
Salmonellosis.
*Dysentery.
An 8-year-old boy fell ill acutely: he presents with fever, weakness, headache, abdominal pain,
recurrent vomiting, then diarrhea and tenesmus. Stools occur 12 times daily, are scanty, contain a lot of
mucus, pus, streaks of blood. His sigmoid gut is tender and hardened. What is your diagnosis?
*Dysentery
Salmonellosis
Cholera
Staphylococcal
gastroenteritis Escherichiosis
A 8-year-old boy fell ill acutely: fever, weakness, headache, abdominal pain, recurrent vomiting, then
diarrhea and tenesmus. Stools occur 12 times daily, are scanty, contain a lot of mucus, pus, streaks of
blood. His sigmoid gut is tender and hardened. What is your diagnosis?
*Dysentery.
Escherichiosis.
Salmonellosis.
Cholera.
Staphylococcal gastroenteritis.
Tuberculin skin test.
A child of 8 years old had a fever of 38.6°С, repeated vomiting, stool 15 times with admixture of
mucus and blood streaks, on examination he was pale, sigmoid intestine is spasmic. Choose optimum
criteria of antibacterial therapy:
*Maximal duration of 5-7 days
Maximal duration is determined by dynamics of clinical manifestations
In absence of the effect during 3 days it is necessary to change the antibiotic
Way of introduction depends on severity of the disease and properties of the
drug All answers are true
A child of 10 years old is ill with dysentery. On the 2-nd day the body temperature was 38.4°С, there
were flaccidity, weakness, loss of appetite, stool was scanty, mucous, with blood, 10 times. The choice
of starting antibacterial therapy in dysentery is determined by:
Form of the disease
Age of the child
Sensitivity of shigella to antibiotics in the given
area Premorbid background
*All answers are true
A child of 12 years old is ill with a severe form of dysentery. Taking into consideration increase of
antibiotic-resistance of the causative agent at the modern stage, it is necessary to use for starting
34
therapy of severe forms:
*Cephalosporins of III generation, as a reserve –
fluoroquinolones Polymixin, as a reserve - levomycetin
Gentamycin, as a reserve -levomycetin
Ampicillin, as a reserve - gentamycin
Nifuroxazid, as a reserve - ampicillin
A baby of 10 months is ill with a severe form of acute Flexner’s dysentery. There are hyperthermia, 6
times repeated vomiting, watery stool with plenty of green mucus and blood streaks 15 times, in
defecation it makes an effort, jerks its legs. What complications can arise?
Infectious-toxic shock
Intestinal bleeding
Peritonitis, invagination
Prolapse of the anus, paraproctitis
*All answers are true
A child of years old is ill with an acute intestinal infection. He has a fever up to 39°С, refuses food and
drinking, vomits up to 5 times, stool is frequent, watery, with admixture of mucus and greens. For
bacteriological examination we use:
Blood
Feces and urine
Lavage of the stomach and intestine
Vomit
*All answers are correct
A child of 8 years old has a syndrome of hemocolitis (frequent watery stool with mucus and blood,
tenderness in the left iliac area) without substantial increase of the temperature. What diseases are
necessary to exclude?
Amebiasis
Balantidiasis
Chronic nonspecific inflammatory diseases of the intestine
Gastrointestinal bleeding as a result of diverticulitis
*All the aforesaid
35
Salmonellosis, a rotavirus infection
Child is 1 year old. Complaints: T-39,8 ° C, liquid stools. Within five days of feverish with periodic
high temperature figures. Sluggish, pale. Turgor pressure was reduced. Crying without tears. Language
is overlaid with a white bloom. Cardiac muted. Swollen abdomen, rumbling in the belly button, there
is tenderness to palpation. Liver + 3,5 cm chair liquid, 10 times, green with slime. Your diagnosis?
Dysentery.
Yersiniosis.
*Salmonella.
Esherihioz.
Rotavirus.
The boy of 11 months old after the eating of not boiled milk is ill for 5 days: temperature is 38-39°С;
he has liquid excrements and vomiting. The child is weak and pallid. Tongue is furred with white
deposit. Heart sounds are muffled. Belly is bloating, rumbling around the belly-button, a liver +3 cm.
Excrements are liquid, up to 5 times a day, has darkly green color with an impurity of slime. What is
the most probable diagnosis?
*Salmonellosis.
Staphylococcal intestinal infection.
Rotaviral infection.
Esherihiosis.
Acute shigellosis.
A 10 month old boy has been ill for 5 days after consumption of unboiled milk. Body temperature is
38-39ºC, there is vomiting, liquid stool. The child is pale and inert. His tongue is covered with white
deposition. Heart sounds are muffled. Abdomen is swollen, there is borborygmus in the region of
umbilicus, liver is enlarged by 3 cm. Stool is liquid, dark-green, with admixtures of mucus, 5 times a
day. What is the most probable diagnosis?
*Salmonellosis
Staphylococcal enteric
infection Escherichiosis
Acute shigellosis
Rotaviral infection
The child of 9 months, did fall ill acutely, in winter, from an increase in the temperature to 39°C,
vomiting, liquid defecations, uneasiness, catarrhal manifestations. It is hospitalized on 2 day of
disease, a serious condition. Repeated vomiting. Excrements watery, discolored, unfaeces, every hour.
Signs of toxic - exsicosis 2 tbsp. In laboratory examination were diagnosed as rotavirus infection.
What is the main method of treatment?
Probiotics.
Dietotherapy.
*Rehydration therapy.
Enzyme preparations.
Antibiotic.
The child of 7 months, became ill in the winter from an increase in the temperature to 38°C, two-fold
vomiting, abundant yellow watery defecations to 10 times in a 24 hour period. What are the leading
mechanism of the pathogenesis of diarrhea?
*Secretory diarrhea.
Invasive diarrhea.
Enteroplegia.
Intussusception.
Anomalies of innervation of the gut.
36
A 10 y.o. child who is at oligoanuretic stage of acute renal insufficiency has got sensations of pricking
in the mucous membrane of oral cavity and tongue, extremities numbness, reduced reflexes,
respiratory disturbance, arrhythmia. What are these symptoms caused by?
*Hyperkaliemia
Hyponatremia
Hyperazotemia
Acidosis
Alkalosis
A child of 4 years old was brought to the clinic with acute dysfunction of the intestine. Signs of
enterocolitis prevail in the clinical course. On the third day blood streaks were found in feces. On the
fifth day the hemolytic-uremic syndrome (Gasser’s syndrome) developed. In what escherichiosis the
hemolytic-uremic syndrome is observed?
Enteroinvasive
*Enterohemorrhagic
Enterotoxigenic
Enteropathogenic
All answers are true
A baby of 6 months was admitted to the clinic with acute dysfunction of the gastrointestinal tract. On
bacteriological examination enteropathogenic E. Coli O111 was isolated. What affection of the
gastrointestinal tract is characteristic of this causative agent?
Stomach
Duodenum
*Small intestine
Large intestine
All answers are true
10 babies fell ill with a dysfunction of the intestine in the somatic postnatal department. A diarrheal
syndrome prevails in the clinical picture. The most reliable method of confirmation of the diagnosis is:
*Bacteriological
Reaction of agglutination Reaction
of indirect agglutination Reaction
of passive agglutination All
answers are true
12 children fell ill with a dysfunction of the intestine in the postnatal department of children's home. A
diarrheal syndrome prevails in the clinical picture. What serovara of escherichia cause the disease in
children more often in the first year of life?
*Enteropathogenic
Enterotoxigenic
Enterohemorrhagic
Enteroinvasive
All answers are true
20 children fell ill with an acute dysfunction of the intestine in the village kindergarten. On
bacteriological examination E. Coli O111 was isolated. Specify the most probable source of infection:
Cattle
Pigs
*Human
Domestic birds
All answers are true
37
A child of 10 years old experiences the signs of arthritis of knee and ankle joints. What causative agent
causes diarrhea with probable development of reactive arthritis?
Campylobacter jejuna
*Jersinia enterocolitica
Salmonella
Shigella
All the aforesaid
A 3-month boy has been having a severe watery diarrhea for 12 hours. Objectively: tissue turgor is
lowered, eyes are sunk down, the mucous membrane of the mouth is dry, oliguria. What causative
agents these symptoms are characteristic of:
Shigella dysenteriae
Enterohemorrhagic E. Coli
Salmonella typhi. Enteroinvasive E. Coli
*Enterotoxigenic E. Coli, Vibrio Cholerae
Helicobacter pylori, Plesiomonas shigeloides
A child fell abruptly ill. Frequent watery stool developed 6 hours ago. The body temperature is normal.
Then vomiting took place. On physical examination: child 's voice is hoarse, eyes are deeply set in the
orbits. The pulse is frequent. Blood pressure is low. There is no urine. What is the preliminary
diagnosis?
Salmonellosis
Dysentery
*Cholera
Toxic food-borne infection
Typhoid fever
Sick M., 2 years, for the third day is found on the treatment apropos of acute enterocolitis.
Antibacterial and detoxification therapy is obtained. State without the worsening. It is objective: the
general state is severe, consciousness is preserved, but darkened. The skin is pale. Pasty, AP -90/60
mm Hg, FCR- of 132 beatings/min, RFR - of 34/min. In the analyses of the blood: Hb of 70 g/l, Er 2,3x1012/l, general. protein 58 g/l, general. bilirubin 18,6 mmole/liter: direct 4,8 mmole/liter, urea 24
mmole/liter, creatinine -140 of mkmole/l. Diuresis 200 ml/day. About what complication of main
disease it is necessary to think?
Hemolytic- uremic syndrome.
Infectious-toxic shock. *Acute
kidney deficiency.
Anhydremical shock.
Septic shock.
Sick A., 3 years, are found on the treatment in the resuscitation department with the diagnosis: Acute
kidney deficiency, oligoanuric stage. On ECG: high T-wave, the expansion of complex QRS, the
displacement of the interval S- T lower than contour isoline. About what disturbance of electrolytic
balance it is possible to think?
*Hyperkaliemias.
Hypokalemias.
Hypocalcemias.
Hypercalcemias.
Hyperphosphatemia.
A baby, aged 2 years, has a syndrome of malabsorbtion. On ECG: S-T segment depression, wave T
inversion and high wave U. What is the cause of these changes?
Hypomagnesaemia.
38
Hyperkalihistia.
Hypercalcihistia.
*Hypokalihistia.
Hypocalcihistia.
At the child of 2 years old with malabsorption syndrome on electrocardiogram was detect depression
of ST-T segment, inversion of T wave and increase of U wave. What does this changes caused by?
*Hypokaliemia.
Hyperkaliemia.
Hypercalcemia.
Hypocalcemia.
Hypomagnesemia.
The girl of 7 years old is in intensive care department with acute renal insufficiency. In the
biochemical analysis of blood the level of potassium is 7 mmol/L. This condition is menacing on
origin of:
*Asystole.
Anorexia.
Anuria.
Hypoxemia.
Hypercapnia.
The girl of 7 years, is situated in the intensive care unit apropos of acute kidney deficiency. In the
biochemical analysis of the blood the level of potassium is 7 mmole/liter. This state threatening on the
appearance:
Anorexia
Anuria.
Hypoxemias.
*Asystolia.
Hypercapnia.
During the first days of the disease at an acute intestinal infection gastroenterocolitis, child 9 months
lost 5% of body weight. How can we eliminate the water - the salt scarcity?
*Assign oral rehydration.
Assign parenteral rehydration for 2
days. Assign a rational diet therapy.
Assign enteros-sorbent.
Assign biopreparations.
39
Viral hepatits Jaundice
With the inspection of children from the focus of virus hepatite A in the 8-years old child in the
absence of clinical manifestations in the blood they are revealed: anti- -HAV - Ig M in the high
concentration, ALT-1,8 mmole/liter. Your diagnosis?
Hepatitis B.
Hepatitis C.
Hepatitis D.
*Hepatitis A.
Hepatitis E.
In the child of 7 years the preliminary diagnosis of acute virus hepatite A was established. What
research will confirm the etiology of disease and the acuteness of process?
Anti- -HAV-IgG, by the method of IFA.
Anti- -HAV-IgM, by the method of IFA.
*Anti- -HAV-IgA, by the method of IFA.
Anti- -HAV-IgE, by the method of IFA.
Anti- -HAV-IgD, by the method of IFA.
The boy of 10 years old has chronic virus hepatitis B with the maximal activity. What of this
laboratory tests can the most precisely characterizes degree of cytolysis?
Crude protein.
Takara-Ara's test.
Prothrombin.
*Transaminases.
Veltmam's test.
The child of 8 years old has clinic of hepatitis. At the examination HBsAg, antibodies of Ig M class to
korovsky antigen were detect. What hepatitis has the child?
*Virus hepatitis B.
Virus hepatitis C.
Virus hepatitis D.
Virus hepatitis Е.
Virus hepatitis A.
At the child of 3 months old icterus and hepatosplenomegaly has appeared after several days of
anxiety, anorexia, low grade fever; urine has dark color, an excrement are decolorized. In the age of 1
month there were hemotransfusions. What is the most probable diagnosis?
Hemolytic anemia.
Biliary atresia. *Virus
hepatitis B.
Conjugation jaundice.
Virus hepatitis A.
In 3-month-old baby with low-grade fever, motor anxiety, anorexia, jaundice, hepatosplenomegaly,
dark urine, discolored stool occurs. The baby has had blood transfusion 2 month ago. What is the most
probable diagnosis?
*Viral hepatits B
Hemolytic anemia
Viral hepatitis A
Atresia of biliferous tructs
Conjugation jaundice
40
A 10-year-old child is sick with chronic viral hepatitis B with marked activity of the process. Total
bilirubin — 70 µmol/L, direct – 26 µmol/L, indirect 44 µmol/L, AST - 6,2 mmol/L, ALT - 4,8
mmol/L. What mechanism underlies the transaminase level increase of this patient?
Intrahepatic cholestasis.
Hypersplenism.
Failure of the synthetic function of the
liver. *Cytolysis of hepatocytes.
Failure of bilirubin conjugation.
A 6-year-old boy is seen in clinic and diagnosed as having mild form of viral hepatitis A. What is the
most important in treatment?
*Regimen and diet
Hepatoprotectors
Antiviral medications
Corucosteroides
Infusion therapy
At boy 10 years old yellow skin and sclera, abdominal pain, nausea, increased liver and spleen
appeared. Previously, due to blunt abdominal trauma was made a blood transfusion. Total bilirubin
42.0 mmol / L, direct - 26 micromol / l, indirect - 12 micromol / l, AST - 4,2 mmol / L, ALT - 3,6
mmol / l were detected RNA and antibody Ig class M hepatitis B virus C. Enter a rational treatment in
this phase of the disease.
Hemosorbtion.
Glucocorticoids.
Infusion Therapy.
*Antivirals.
Intestinal chelators.
The boy of 8 years year ago was injured with hepatitis B. In the recent two months he complains about
the increased fatigue, the disturbance of sleep, worsening in the appetite, nausea, especially in the
morning. Integuments without jaundice, liver and spleen are palpated 1 cm 1 cm below the costal edge,
painless. Activity of ALT is 2.2 mmol / liter. This state can be estimated as:
Relapse-hepatitis B.
Dyskinesia of biliary-tract.
Residual effects of endured hepatitis.
*The development of chronic hepatitis.
The development of cirrhosis.
Children of 3 months. He has been restless during the week, regurgitate, refused food, the body
temperature is subfebrile. Two days ago, appeared dark urine and discolored feces, then - jaundice.
Liver and spleen were enlarged. In the neonatal period the child received a blood transfusion. What is
the most likely diagnosis?
*Viral hepatitis C.
Viral hepatitis A.
Hemolytic anemia.
Conjugate jaundice.
Atresia biliary tract.
4-year-old patient was admitted to the intensive care unit with hemorrhagic shock due to gastric
bleeding. He has a history of hepatitis B during the last 5 years. The source of bleeding is esophageal
veins. What is the most effective method for control of the bleeding?
Intravenous administration of
pituitrin. Administration of plasma.
41
Operation.
*Introduction of obturator nasogastric
tube. Hemostatic therapy.
Jaundice
The skin of 3 days old child has got yellow color. The child was born with weight 3.200 kg, length of a
body 52 sm. active. There is puerile respiration above lungs. Breathing rate - 36 in 1 minute, heart
sounds are rhythmical. The heart rate - 130 impacts in a minute, Belly soft. The liver + 2 sm, lien is not
palpated. Excrements in the form of meconium. What is the most probable diagnosis?
Hemorrhagic disease of
newborn. Sepsis of newborns.
*Physiological icterus.
Atresia of the bile passages.
Minkowsky-Shauffard disease.
The child has a 3 day life has become icteric skin color. Child was born with a weight of 3,200 kg,
body length 52 cm Active. Above Light puerilnoe breath. BH - 36 in 1 min. Cardiac rhythmic. Heart
rate - 130 in 1 min. Abdomen soft. The liver appears from under the costal arch to 2 cm spleen was not
palpiruetsya. Excretions are in the form of meconium. The most likely diagnosis:
*Pedicterus.
Hemolytic disease of
newborn. Neonatal sepsis.
Anemia-Minkowski-Shafar.
Atresia of bile duct.
In reported newborn from 3 through 10 days of life jaundice was observed. The general condition
remained satisfactory. The maximum level of bilirubin in the blood at that time was 102 mmol/l, of
which 8.2 mmol/l at the expense nonconjugated. Development of a what state is most probably this
child?
Fetal-hepatitis.
Hemolytic disease of
newborn. *Pedicterus.
Hereditary icterohemolytic
anemia. Atresia, biliary tract.
The child of six months old since the birth has icterus with greenish shade. The manifestations of
hemorrhagic diathesis and dermal itch are observed. What is the most probable pathology at the child?
Gilbert's syndrome.
Dabin-Johnson's
syndrome. *Biliary atresia.
Crigler-Najjar syndrome.
Erythroblastosis.
A neonate was born from the 1st gestation on term. The jaundice was revealed on the 2nd day of life,
then it became more acute. The adynamia, vomiting and hepatomegaly were observed. Indirect
bilirubin level was 275 mol/L, direct bilirubin level - 5 mol/L, Hb - 150 g/l. Mother’s blood group 0[I], Rh+, child’s blood group- A[II], Rh+. What is the most probable diagnosis?
*Hemolytic disease of the neonate [АВО incompatibility], icteric type
Jaundice due to conjugation
disorder Hepatitis
Physiological jaundice
42
Hemolytic disease of the neonate [Rh - incompatibility]
A baby boy was born in time, it was his mother's 1st pregnancy. The jaundice was revealed on the 2nd
day of life, then it progressed. The adynamia, vomiting and hepatomegaly were presented. The indirect
bilirubin level was 275 mcmol/L, the direct bilirubin level - 5 mcmol/L, Hb- 150 g/L. Mother's blood
group - 0(I), Rh+, child's blood group - A(II), Rh+. Make a diagnosis.
*Hemolytic disease of newborn (АВО incompatibility), icteric type
Jaundice due to conjugation
disorder Hepatitis
Physiological jaundice
Hemolytic disease of newborn (Rh - incompatibility)
A full-term baby (the 1st uncomplicated pregnancy, difficult labour) had a cephalogematoma. On the
2nd day there was jaundice, on the third the following changes in neurological status appeared:
nystagmus, Graefe syndrome. Urine was yellow, feces were of golden-yellow colour. Mother's blood
group is A (II) Rh-, the baby's one - A (II) Rh+. On the third day the child's Hb was 200g/l, RBCs 6,1\cdot10^{12}/l, blood bilirubin - 58 micromole/l at the expense of unbound fraction. What caused
the jaundice in the child?
*Craniocerebral birth
trauma Physiological
jaundice Neonatal anaemia
Biliary atresia
Fetal hepatitis
43
Flu
A child of 3 years old had sharply elevated body temperature, there was a repeated vomiting, the child
was restless. On examination: mucous membrane of the fauces was hyperemic, there were no coatings.
The tremor of the extremities, increased tendon reflexes, rigidity of the occipital muscles were marked.
In liquor: cytosis - separate cells, protein - 0.33g/l. On the 4-th day the condition improved, meningeal
signs disappeared. What is your diagnosis?
Tuberculous meningitis
Poliomyelitis, meningeal form
Encephalitis of viral etiology
Enteroviral meningitis
*Flu, meningism
A 3 y.o. girl has had a temperature rise up to $38^0С$, rhinitis, dry superficial cough, flabbiness,
appetite loss. Palpation didn't reveal any changes over her lungs. Percussion sound has a wooden
resonance, auscultation revealed puerile breathing, no rales. In blood: leukopenia, lymphocytosis,
increased ESR. What is the most probable diagnosis?
*Acute simple tracheitis
Acute obstructive bronchitis
Recurrent bronchitis, acute condition
Acute simple bronchitis
Bilateral microfocal pneumonia
Ambulace brought to the hospital a patient with acute respiratory viral infection. The illness began
suddenly with temperature rise up to 39,90С. He complains of headache in frontotemporal lobes, pain
in eyeballs, aching of the whole body, nose stuffiness, sore throat, dry cough. At home he had a nasal
hemorrhage twice. What type of acute respiratory viral infection is it?
*Influenza
Adenoviral infection
Parainfluenza RSinfection Enterovirus
infection
A patient, aged 16, complains of headache, mainly in the frontal and temporal areas, superciliary arch,
appearing of vomiting at the peak of headache, pain during the eyeballs movement, joint's pain. On
examination: excited, t0- 390С, Ps- 110/min. Tonic and clonus cramps. Uncertain meningeal signs.
What is the most likely diagnosis?
*Influenza with cerebral edema
manifestations Adenovirus infection
Respiratory syncytial virus
Influenza, typical disease
duration Parainfluenza
Child is 5 days. From І full-term pregnancy in the second half of which my mother suffered a flu. On
examination, marked reduction of motor activity, pale skin, and periorbitalny perioral cyanosis, which
is reinforced by sucking the breast, anxiety, ―puffing‖, the retraction of inter-rib spaces. Percussion
revealed an increase in relative cardiac dullness borders on all sides, auscultatory-tachycardia, heart
rate of 170 in 1 minute, heart tones are weakened. Tachypnea. Hepatomegaly. On x-ray of the chest
there is increased the size of the heart. The ECG are recorded signs of myocardial hypertrophy of the
ventricles, persistent violations of rhythm and conduction. Which of the following is the most likely
cause of such changes?
*Non-rheumatic
myocarditis. Fibroelastoz.
44
Ductus arteriosus.
Constrictive myopericarditis.
Infective endocarditis?
The patient of 10 years, in whom on 2nd day of disease are observed the symptoms of the influenza of
average gravity, obtains aspirin, gluconate of calcium; naphthazoline 0,1% in the drops into the nose.
What antiviral preparation it is expedient to appoint?
*Remantadin.
Laferon.
Acyclovir.
Ribavirin.
Herpevir.
Parainfluenza virus
A child is 2 years old. The child complains of hoarse voice, dyspnoea with obstructed inspiration. The
disease started 3 days ago from dry cough and nose stuffiness. Objectively: general condition is
unbalanced, stridor is present. The child's skin is pale. Body temperature is 37,7ºC. The palatine arches
are hyperemic. There is no deposit. Heart sounds are rhythmic. Auscultation of lungs reveals rough
breathing sounds, crepitation is absent. Parainfluenza virus has been detected in nasopharynx lavage.
What is the most likely diagnosis?
*Acute laryngotracheitis
Epiglottitis
Foreign body
Diphtheria
Laryngospasm
The girl of 4th years old became ill suddenly. The voice became hoarse, sometimes aphonic. There
was coarse, sonorous, dry, unproductive tussis. Body temperature was 37,4°С. Objectively: the child is
quiet; position in bed is compelled (with the fixed shoulder girdle). Stenotic respiration is well audible
on distance. It is precisely visible a permanent job of respiratory musculation. Auscultation: respiration
is rigid, even depressed, especially in back and bottom departments of lungs. What mechanism
dominates over a pathogenesis of a croup?
*Edema of a mucosa of larynx and trachea. Spasm
of unstriated muscles of larynx and trachea.
Obstruction.
Laryngospasm.
Hypersecretion of glands of mucosa of a larynx, trachea and bronchi.
An infant aged 1 year on the third day of common cold at night developed inspiratory stridor, hoarse
voice and barking cough. Physical examination revealed suprasternal and intercostals chest retractions.
There is a bluish skin discoloration. Moistly seen over the upper lip. The respiratory rate is 52 per min
and pulse - 122 bpm. The body temperature is 37,5°C. What disease does the infant have?
Acute bronchiolitis with respiratory
distress. Bronchopneumonia without
complications. Acute epiglottitis.
*Acute infectious croup due to viral
laryngotracheitis. Acute laryngitis.
Child of 10 months. It fell ill acutely: increased the temperature of body to 39ºC, appeared cough and
runny nose. On day 2 of disease at night the child suddenly became restless, appeared the rough,
45
―barking‖ cough, hoarseness of voice, inspiratory shortness of breath. About what illness it is possible
to think?
Diphtheria.
Pertussis.
Intraorbital foreignbody. *Paragrip.
TB bronhoadenit.
A 1 year-old baby fell ill abruptly fever up to 38ºC, coryza, cough. Hoarseness of voice, dry barking
cough, stenotic breathing and motor anxiety occur at night. What pathogen caused this disease?
Adenovirus
Corynebacterium
diphtheria *Parainfluenza
virus Rhinovirus
Reovirus
A 9-month-old baby has spinal form of poliomyelitis. What group of viruses does the causative agent
of the disease belong to?
*Picornaviruses
Paramyxoviruses
Togoviruses
Adenoviruses
Rotaviruses
A baby of 9 months fell ill sharply: the temperature rose up to 38.8°C, there was a cough, hoarse voice.
In some hours the condition sharply aggravated, symptoms of laryngotracheitis with stenosis of I-II
degree developed. What of causative agents has most likely caused croupe in the baby?
Virus of flu
*Virus of paraflu
Adenovirus
Enterovirus
Rhinovirus
Child 2,5 years old, fell ill with acute illness started with a barking cough, moderate secretions from
the nose, temperature 38°C. In the first day of the disease the child awoke at night because of rough
cough, shortness of breath. Shortness of breath is of inspiratory nature, is observed as the excitation,
and in rest. Breathing involved supporting musculature. Is noted the blurry retraction of the yielding
places of chest and epigastrium, perioral cyanosis, pallor, and tachycardia. The child immunized by
age. What is the best possible preliminary diagnosis?
*ARI, acute laryngotracheobronchitis, stenosis of 2
degree. Diphtheria-respiratory.
Intraorbital foreign-body
airway. Asthma.
TB intrathoracic lymph nodes.
46
Acute respiratory viral infection and complications
The child of 14 years old has complained of weakness, temperature rise up to 37,8°С, mucous
discharge from a nose, and pharyngalgia during swallowing, feeling of "sand" in eyes during 7 days.
Objectively: the enlargement of backcervical and submandibular lymph nodes, edema and injection of
vessels of conjunctivas, hyperemia of mucosa of guttur, hypertrophy of tonsils, no incrustation. What
is the most probable diagnosis?
Flu.
Parainfluenza.
Infectious mononucleosis.
*Adenoviral infection.
Rhinoviral infection.
A 3 m.o. child fell seriously ill, body temperature rised up to $37,8^0C$, there is semicough. On the 3rd day the cough grew worse, dyspnea appeared. On percussion: tympanic sound above lungs, on
auscultation: a lot of fine moist and wheezing rales during expiration. What is the most probable
diagnosis?
*Acute respiratory viral infection, bronchiolitis
Acute respiratory viral infection, bronchopneumonia
Acute respiratory viral infection, bronchitis
Acute respiratory viral infection, bronchitis with asthmatic component
Acute respiratory viral infection, focal pneumonia
A 3 month old infant suffering from acute segmental pneumonia has dyspnea (respiration rate - 80 per
minute), paradoxical breathing, tachycardia, total cyanosis. Respiration and pulse - ratio is 1:2. The
heart dullness under normal size. Such signs characterise:
*Respiratory failure of III
degree Respiratory failure of I
degree Respiratory failure of II
degree Myocarditis
Congenital heart malformation
A child was born at a gestational age of 34 weeks in grave condition. The leading symptoms were
respiratory distress symptoms, namely sonorous and prolonged expiration, involving additional
muscles into respiratory process. The Silverman score at birth was 0 points, in 3 hours it was 3 points
with clinical findings. Which diagnostic study will allow to diagnose the form of pneumopathy?
*X-ray of chest
Clinical blood test
Determination of blood gas composition
Proteinogram
Immunoassay
The 10 y.o. boy has complains on headache, weakness, fever 40ºС, vomiting, expressed dyspnea, pale
skin with flush on right cheek, lag of right hemithorax respiratory movement, dullness on percussion
over low lobe of right lung, weakness of vesicular respiration in this zone. The abdomen is painless
and soft at palpation. Which disease lead to these symptoms and signs?
*Pneumonia
croupousa Intestinal
infection Acute
appendicitis Acute
cholecystitis Flu
An 18-month-old child was taken to a hospital on the 4-th day of the disease. The disease began
47
acutely with temperature 39, weakness, cough, breathlessness. He is pale, cyanotic, has had febrile
temperature for over 3 days. There are crepitative fine bubbling rales on auscultation. Percussion
sound is shortened in the right infrascapular region. X-ray picture shows non-homogeneous segment
infiltration 8-10 mm on the right, the intensification of lung pattern. Your diagnosis:
*Segmentary
pneumonia Grippe
Bronchitis
Bronchiolitis
Interstitial pneumonia
A 6 week old child is admitted because of tachypnea. Birth had been uneventful, although
conjunctivitis developed on the third day of life and lasted for about 2 weeks. Physical examination
reveals tachypnea, bilateral inspiratory crackles and single expiratory wheezing. Bilateral pneumonia
is evident on chest X-ray. The child is afebrile and has no history of fever. White blood cell count is
15cdot109/l, with 28% of eosinophils. The most likely cause of this child's symptoms is:
*Clamydia trachomanis
Pneumocystis carinii
Mycoplasma
pneumoniae Visceral
larva migrans Varicella
A 4 year old girl was playing with her toys and suddenly she got an attack of cough, dyspnea.
Objectively: respiration rate - 45/min, heart rate - 130/min. Percussion revealed dullness of percussion
sound on the right in the lower parts. Auscultation revealed diminished breath sounds with bronchial
resonance on the right. X-ray picture showed shadowing of the lower part of lungs on the right. Blood
analysis revealed no signs of inflammation. The child was diagnosed with foreign body in the right
bronchus. What complication caused such clinical presentations?
*Atelectasis
Emphysema
Pneumothorax
Bronchitis
Pneumonia
A 3-year-old child was playing in a playpen when he suddenly developed paroxysmal cough and
shortness of breath. Objectively: dry cough, mixed dyspnoea. Lung auscultation revealed some
wheezes. Breathing sounds on the right are diminished. The child doesn't mix with other children.
Immunization is age-appropriate. What pathological condition can be suspected?
*Foreign body in the respiratory
tracts Pneumonia
Acute respiratory viral
infection Pertussis
Bronchial asthma
The 7 m.o. infant is suffering from acute pneumonia which was complicated by cardiovascular
insufficiency and respiratory failure of II degree. The accompanied diagnosis is malnutrition of II
degree. Choose the best variant of therapy:
*Ampiox and Amicacin
Macropen and Penicillin
Penicillin and Ampiox
Gentamycin and Macropen
Ampiox and Polymixin
48
A baby, aged 8 months, is examined and a diagnosis of atypical community-acquired pneumonia of
chlamidial etiology is made. What is the optimum alternative of antibiotic therapy in this case?
Aminopenicillin.
Cephalosporin of the II generation.
*Macrolide antibiotic of the II
generation. Aminoglycoside antibiotic.
Macrolide antibiotic of the I generation.
A 8-month-old child is diagnosed atypical community-acquired Chlamidia trachomatis pneumonia.
Select the best antibiotic.
Macrolide of 1st generation.
Aminopenicilline. *Macrolide
of 2nd generation.
Cephalosporine of 2nd
generation. Aminoglycoside.
The child of 8 months old after corresponding examination has such diagnosis as atypical communityacquired pneumonia of clamydion etiologies. Choose an optimum variant of an antibioticotherapia:
Cephalosporin of second generation.
Aminoglycoside.
Aminoglycosides.
*Macrolide of second generations.
Macrolide of first generations.
A 1-year-old infant is admitted for failure to thrive. During the neonatal period he had an exploratory
laparotomy for intestinal obstruction. At 3,8 and 11 month of age, he had respiratory infections
diagnosed as bronchitis. Physical examination: weight of 6,8 kg, thin extremities with very little
subcutaneous tissue, and a protuberant abdomen. The essentials diagnostic study in this child is:
Skin test for milk allergy.
Bronchoscopy.
*Sweat electrolytes. Serum
immunoglobulin le
In child with the hereditary hemolytic anemia against the background of ARD, high fever the ictericity
of the skins and mucosa appeared, increased the liver and spleen. What complication of hemolytic
anemia is threatening for the life of child?
Bilirubinic encephalopathy.
Hepatic insufficiency. Heart
insufficiency.
*Haemolytically-uremic syndrome.
Hyperthermal syndrome.
he child is 11 years. The complaints of general weakness, rapid fatigue, sweating, anorexia, weight
loss, presence of rash, pain in the heart, joints, muscles, bones, increased body temperature to
subfebrile digits, with short-term rises to 39-40 ° C daytime or evening with a fever and subsequent
perspire (spreads of temperature during the day more than 1,5 ° C). On examination, there is pale skin
with a grayish tinge. At sites shins, forearms, neck, elbow, lateral surfaces of the trunk, oral mucosa,
transitional age folds is visible small hemorrhagic rash. Finite phalanx of fingers are in the form of
drum sticks, nail-type time stack. Auscultation of the heart auscultated protodiastolic "sawing" noise in
the IV intercostal space near the left edge of the sternum, which is worse on inspiration. Which of the
following diseases most likely have a child?
Rheumatism.
Congenital carditis.
49
Non-rheumatic carditis.
*Infective endocarditis.
Systemic lupus erythematosus.
In the 3-year-old child after AVR deteriorated general condition, appeared quick fatigue. Skin pale,
enlarged left heart border, I tone is deaf on the top, soft systolic sound over the top. On ECG signs of
left ventricular overload. What is the most likely diagnosis?
*Non-rheumatic myocarditis.
Rheumatic carditis.
Congestive Cardiomyopathy.
Congenital carditis. Acquired
heart disease.
Girl 9 years old in 2 months of age suffered pneumonia, in 3 years with severe measles. Underwent a
serious flu with high fever. Within 3 days against the backdrop of fever felt fear, was restless, saw
devils "with a nasty snouts, which something spoke. After lowering the temperature and improve wellbeing on day 7 went to school, but quickly tired, headache, 4 days ago saw on the wall to wall pictures
snakes, cockroaches, there were "voices" again became restless and therefore was hospitalized in
psychiatric hospital. What is it need to be assigned a patient to prevent re-delirious state of infectious
origin?
Detoxification therapy.
Antidepressant.
Restorative therapy.
*Tranquilizers.
Nootropics
A child 12 years diagnosed with non-rheumatic carditis, acute, moderately severe, with signs of CI of I
degree in 7 days after acute respiratory viral infection. What is the pathogenetic mechanism is the basis
of the disease?
Infection.
Allergic.
Autoimmune.
*Infectious-allergic.
Toxic.
In 10 years old girl after ARVI the complaints of pain in the heart and shortness of breath at the
physical load appeared. During inspection: the skin is pale, the increased humidity. The left measure of
heart is displaced to the mediaclavicular line, heart tones are weakened, there is tender systolic noise
above the top. FCR-124 per minute, AP-90/60 mm Hg. Blood: anemia of I-st grade, moderate
leukocytosis, eosinophilia, the proteins of acute phase are in norm. ECG: sinus tachycardia, the
disturbance of the processes of repolarization, the displacement of the interval ST lower than contour
isoline. The described symptoms are characteristic for:
*Nonrheumatic carditis.
Pancarditis.
Septic endocarditis.
Fibroelastosis.
Rheumocarditis.
In a family of 5 persons (3 children) there is sick child of 11 months. Appeared lethargic, then raised
the body temperature to 39 ° C, then - nasal congestion, repeated vomiting. Objective: a serious
condition, and occasional hemorrhage in the neck, palate mucosa, sclera. Throat is cyanotic, there is
grainity o posterior wall of the pharynx. From the nose - scarce mucous discharge. Breathing is hard,
46 per minute. Pulse - 156 per minute. Cardiac is muted. What is the primary measure is held in the
50
hearth?
*Isolation patient in Isolation unit.
Hospitalization of the patient in a somatic
department. The final disinfection.
Antibiotic.
Introduction of contact-immunoglobulin.
The boy of 12 years old had acute respiratory disease for 5 days. Local pediatrician detect an
arrhythmia at examination of this child. At auscultation: weak first cardiac sound on apex short
localized systolic hum in 5 point. On an electrocardiogram - extrasystole, depression of amplitude of T
wave. What is the most possible diagnosis?
*Infection-allergic myocarditis.
Rheumatic myocarditis.
Vegetovascular dystonia.
Bacterial endocarditis.
Functional extrasystole.
The boy of 5 years old who receives ampicillin for treatment of ARD, for 5th day of treatment signs of
intoxication had rised, faces became oedematous and joint pain appeared. On skin of a trunk we can
see miliary rash. BP-140/90 mm hg. In next day he had excrete 2100 ml of urine (had received 2000
ml of liquids ). In the biochemical analysis of blood: creatinine- 0,22 mmol/L, urea- 11,8 mmol/L,
potassium- 3,8 mmol/L, sodium -125 mmol/L. In the analysis of urine: proteinuria- 0,99 g/l,
erythrocyturia, eosinophilic leukocyturia. Relative density of urine during the day is 1002 - 1010.
What is the most possible diagnosis?
*Acute interstitial nephritis.
Acute pyelonephritis.
Acute glomerulonephritis with nephritic syndrome.
Acute
glomerulonephritis
with
nephrotic
syndrome. Dysmetabolic nephropathy.
The child of 10 years old complains of reddening of eyes, lacrimation, moderate purulent discharges
from conjunctival cavity, sensation of foreign body in eyes. Objectively: hyperemia of palpebral
conjunctiva. There is conjunctival injection on eye-bulbes. A cornea is transparent. Pupils are 3 mm in
diameter and have quick light response. The lens and vitreous are transparent. An eyeground is in
norm. What is the most probable diagnosis?
Allergic conjunctivitis.
Acute iridocyclitis.
*Acute bacterial conjunctivitis.
Honoblenorrhea.
Adenoviral keratoconjunctivitis.
51
Vaccination
A child born of normal birth. In the mother stated HbsAg carrier. What is the tactics of a doctor
relatively to child's immunizations against hepatitis B?
*Vaccination against hepatitis B in the first 12 hours of
life. Hepatitis B vaccination be postponed for 1 month.
Hepatitis B vaccination be postponed for 6 months.
Hepatitis B vaccination be postponed for 1 year.
Vaccination against hepatitis B is not carried.
Neonate is 5 days old. What vaccination dose of bacillius Calmette-Guerin (bCG) vaccine is necessary
for this child?
0.1 Mg.
0.025 Mg.
0.075 Mg.
*0.05 Mg.
0.2 Mg.
A neonate is 5 days old. What vaccination dose of BCG vaccine (in mg) is necessary for vaccination of
this child?
*0,05 mg
0,025 mg
0,075 mg
0,1 mg
0,2 mg
In the inhabited locality there is an increase of diphtheria during the last 3 years with separate
outbursts in families. What measure can effectively influence the epidemic process of diphtheria and
decrease the morbidity with diphtheria to single cases?
Revelation of carriers.
*Immunization of the
population. Early diagnostics.
Hospitalization of patients.
Disinfection in disease focus.
The boy of 6 years, fell ill with measles. In the family there is another child of 8 months. What basic
counterepidemic measure must be taken relative to contact child?
Carry-active immunization against
measles. Assign antivirals.
*Inject 3 ml of normal human
immunoglobulin. Hospitalized in the hospital.
Antibiotics.
On the second day after preventive vaccination a 2-year-old boy presented with abdominal pain
without clear localization, body temperature rose up to 38ºC. On the third day the child got red papular
haemorrhagic eruption on the extensor surfaces of limbs and around the joints. Knee joints were
edematic and slightly painful. Examination of other organs and systems revealed no pathological
changes. What is the most likely diagnosis?
*Haemorrhagic vasculitis
Thrombocytopenic
purpura Meningococcemia
Urticaria
DIC syndrome
52
A 12 year old girl complains about abrupt weakness, nausea, dizziness, vision impairment. The day
before she ate home-made stockfish, beef. Examination revealed skin pallor, a scratch on the left knee,
dryness of mucous membranes of oral pharynx, bilateral ptosis, mydriatic pupils. The girl is unable to
read a simple text (mist over the eyes). What therapy would be the most adequate in this case?
*Parenteral introduction of polyvalent antibotulinic
serum Parenteral disintoxication
Parenteral introduction of
antibiotics Gastric lavage
Parenteral introduction of antitetanus serum
The boy of 4 months old in 15 mines after the second inoculation by diphtheria and tetanus toxoids and
pertussis vaccine had signs of a Quincke's edema. What preparation should be used for emergency
action?
Adrenaline.
Furosemide.
Heparin.
*Prednisolone.
Seduxen.
Child 2 years old, sick for 2 weeks, coughing within 7 days, the temperature was subfebrile. Then the
coughing attacks of the character got to facial flushing, reprises, a discharge of a viscous, glassy
phlegm up to 15 times a day. Hemorrhages in the sclera of both eyes, bridle language are sore. In the
lungs there are severe breathing. The introduction of what drug can prevent this disease?
*DTP vaccine.
Immunoglobulin.
Antibiotic.
Sulfanilamide.
Bacteriophage.
15 years old schoolboy, roller-skate in the court of school, fell and wounded the skin in the section of
knee joint. Last inoculation by ADCS to him was made at the age of 5 years. What immunization must
be carried out?
*Tetanum anatoxin.
ADCT.
ADT.
ADT- M.
Antitetanus serum.
At boy of 10 months 15 minutes after the second inoculation by vaccine ACDT fixed the signs of
Quincke's edema. What preparation you do use for the rendering to the child of pressing aid?
*Prednisolone.
Heparin.
Papaverine.
Curantil.
Seduxen.
A child aged 10 months to reduce the temperature paracetamol syrup was given, after 2 hours the
temperature was not decreased, and mother gave the child an aspirin at a dose of 0.1 g. After 20
minutes, at child appeared face edema, papular rash, shortness of breath. From history we know that
the boy's grandfather did not tolerate aspirin. If this is Quincke's edema, it is the fact that necessary to
appoint to child first of all?
53
*Glucocorticoids + antihistamine in /
v. Analgin-in / m.
Eufillina-in / v.
Cardiac glycosides.
Antihistamines-enterally.
In child 1 of year with fibroelastose against the background acute respiratory disease suddenly
appeared the uneasiness, acrocyanosis, pulse 132 in1 min, respiratory rate 50 into 1 min, bubbling
moist rales in the lower divisions, RO2 of 60 mm Hg, pO2 60 mm Hg, pCO2 55 mm Hg At X-ray:cardiomegaly, strengthening of pulmonary figure, roots is in the form butterfly wings. Worsening in
the state is connected with:
*Pulmonary edema.
Bronchiolitis.
Bilateral pneumonia.
Angioedema.
Lung abscesses.
A 16-year-old adolescent was vaccinated with DTP. In eight days there was stiffness and pain in the
joints, subfebrile temperature, urticarial skin eruption, enlargement of inguinal, cervical lymph nodes
and spleen. What kind of allergic reaction is observed?
*Immunocomplex.
Hypersensitivity of delayed
type. Cytoxic.
Hypersensitivity of immediate type.
HIV AIDS
In boy from the asocial family of the patient with hemophilia of against the background of generalized
lymphoadenopathia and recurrent candidiasis of oral cavity developed pneumocystic pneumonia, the
correlation coefficient of T- helpers to T-supressors was substantially reduced. What is most possible
the reason for the demonstration of opportunistic infections in this patient?
The chronic disease of the system of the organs of the
digestion. Transitory immunosuppression dependent on age.
Hemophilia.
*VIH- infection, AIDS.
Social-welfare problems
By what way cannot occur the transmission of HIV from mother to
child? During pregnancy
During delivery
Through breast milk
*Aerial-droplet way
All answers are right.
Passively acquired maternal HIV antibodies may be present in the child’s blood until:
A. 6 months of age
12 months of age
16 months of age
*18 months of age
54
24 months of age
Major pulmonary illnesses in children with
AIDS: Streptococcal pneumonia
Bronchial asthma
*Pneumocystis pneumonia
Adenoviral pneumonitis
Emphysema
Major gastro-intestinal illnesses in children with
AIDS: Rotaviral gastroenteritis
*Unexplained chronic
diarrhea Acute duodenal ulcer
Phlegmonous pancreatitis
Constipation
The HIV refers to the
family: Picirnoviridae
Herpesviridae
*Retroviridae
Enteroviridae
Paramixoviridae
An antiretroviral drug
is: *Lamivudine
Rimantadine
Aciclovir
Ribaverini
Zanamivir
The source of infection of AIDS is:
*An ill patient
An ill animal
An ill patient and ill
animal Insect
Is unknown
The main in pathogenesis of AIDS
is: Depression of T-cells
Depression of B-cells
Stimulation of humoral immunity
Depression of thrombocyc
*All answers are right.
Congenital AIDS includes:
Growth retardation Recurrent
bacterial infections
Neurological dysfunction
Dysmorphic syndrome
*All answers are right
The patient of 17 years old complains of liquid excrements during 2 months, body weight loss on 13
kg, weakness, constant subfebrile temperature, and recurrent herpes. Objectively: herpetic rashes on
55
lips, generalized lymphadenopathy, enlargement of liver up to 2 sm. In blood test: red blood cells 4,4х10/12/l, leucocytes - 10,0х10/9/l, e.-2 %, b.-6 %, s.-61 %, l.-17 %, m.-3 %. atypical lymphocytes 6 %. What is the most probable diagnosis?
Infectious mononucleosis.
*AIDS.
Lymphogranulomatosis.
Amebiasis.
Prolonged dysentery.
Pneumonia of pneumocystic etiology, average gravity, RI of 2 stage is diagnosed in 3 monthly child.
From what preparation it is necessary to begin treatment?
*Biseptol.
Penicillin.
Cefalexin.
Lincomycin.
Erythromycin.
The child at the age of 2 weeks, who was born with a gestational age 34-35 weeks was diagnosed
generalized candidiasis. Which of these drugs is the preparation of choice for causal treatment?
Nystatin.
Levorin.
Metronidazole.
*Diflucan.
Klaforan.
TORCH-infections
What infection belongs to the group TORCHinfections? Lues
*Herpetic infection
Measles
Tuberculosis
Scarlatina
What variant of disease in child is the most probably if Toxoplasma infection happened in the first
trimester of pregnancy?
*Miscarriage Mononucleosislike syndrome Asymptomatic
infection
Classical manifestation of acute
disease Mental retardation
In which period of pregnancy is Toxoplasma infection most dangerous for
fetus? *First trimester
Latter half Third
trimester Second
trimester
Intranatal
56
What animals are the basic sources of Toxoplasma
infection? Pigs
Poultry
*Cats
Freshwater
fish Deer
In which period of pregnancy does Toxoplasma infection have minimal risk of affection of
fetus? First trimester
Latter half
*Third trimester
Second trimester
Intranatal
What variant of disease of children is the most probable if Toxoplasma infection happened in the third
trimester of pregnancy?
Miscarriage Mononucleosislike syndrome
Classical manifestation of acute
disease *Asymptomatic infection
Mental retardation
Most frequent form of CMV
infection: *Latent
Encephalitis
Trombocitopenia
Hypotrophy
Chronic infection
A child of 3 years old has been in hospital for 10 days for herpetic gingivostomatitis. Today the child’s
condition is satisfactory, he does not have a fever, eats well, the mucous membrane of the fauces is
pink and clean. Erosions are epithelized. When can the child attend children's group?
In of 2 weeks after discharge
*After clinical recovery
In 22 days after the onset of the
disease In 7 days after discharge
In 12 days after the onset of the disease
A child of 5 years old suffering from allergodermatosis had elevated temperature up to 39.5°С,
flaccidity, vomiting; plentiful vesicular rash developed at the sites affected by dermatosis, it was filled
with transparent content. Enlarged regional lymph nodes were palpated. In 2 days vesicular content
became turbid, umbilicate impression appeared in the centre. The child’s mother suffers from recurrent
herpes of the skin. What disease should we think of?
Neurodermitis
*Caposi herpetiform eczema
Zoster
Stevens-Johnson's
syndrome Chicken-pox
The typical form of localized congenital herpes simplex infection:
Liver impairment
Renal failure
57
*Skin and mucus lesions
Encephalitis
Meningitis
The most frequent form of CNS herpes simplex infection in children
is Meningitis
*Encefalitis
Neurotoxicosis
Encephalopathy
Polyneuritis
What antibiotics do you prefer at the treatment of neonatal sepsis (empirical
therapy)? Penicillin + gentamycin.
Cefazolin + Ampicillin.
Fortum + Amikacin.
*Zinacef + Netromycin.
Norfloxacin + Metragil.
Sick child who had typhoid, on 14-th day of diseases has suddenly feel stomachache. The child is in
consciousness. Pain is acute. The belly takes part in the act of respiration, is not sweel. On the
palpation: the moderate muscle tension of a forward abdominal wall, it is more pain in the right
inguinal region. Hepatic dullness is not determined. The peristalsis is auscultated, but it's flaccid.
Blumberg's sign is positive. Body temperature is 38,9°С. Pulse - 104 impacts in a minute. What is the
most probable diagnosis?
Perforation of an acute gastric
ulcer. Adhesive intestinal
obstruction. Acute pancreatitis.
Acute perforated appendicitis.
*Perforation of a typhoid ulcer.
The flash of food poisoning was registered in urban village. The diagnosis of botulism was put on the
basis of a clinical picture of disease. What of the listed products are necessary for selecting, first of all,
on the analysis for verification of diagnosis?
*Canned food.
Pasteurized
milk. Cabbage.
Boiled meat.
Potato.
The girl of 12 years old complains of weakness, nausea, giddiness, disturbance of vision. She ate
home-brewed dried fish and beef on the day before. At examination we can see the pale skin, scratch
of the left knee, dryness of guttur's mucosa, bilateral ptosis, pupils are dilated. Girl is not unable to
read a simple text ("grid", "fog" before eyes). What therapy will be the most adequate in this case?
*Parenteral administration of polyvalent anti-botulism serum.
Parenteral deintoxication.
Parenteral administration of antibiotics.
Gastric lavage.
58
Parenteral administration of antitetanic serum.
The purulent discharges from the umbilical wound of newborn are marked; a skin around the navel is
tumescent .Objectively: pallid skin with a yellow-grey tint, generalized hemorrhagic rash. The body
temperature is hectic. What is the most probable diagnosis?
*Sepsis.
Hemorrhagic disease of
newborn. Thrombocytopathy.
Omphalitis.
Hemolytic disease of newborns.
The child of 12 years old was hospitalized with an intestinal obstruction. During operation it was
found out that obstruction of small bowel was caused by ball of worms. What worm was it? What it
was a helminth?
Cysticercus.
*Ascarid.
Filaria.
Pinworm.
Hymenolepis nana.
Determine the preparation, which does have a effectiveness for treatment enterobioses and ascariasis
in children and is assigned single-time:
*Pyrantel.
Vermoks.
Piperazine adininat.
Naftamon.
Ditianazin.
What preparation is effective for treatment of enterobiasis and ascaridiasis of children and it is
prescribe for a one taking?
*Pyrantel.
Vermox. Pyperasini
adinatis. Naftamon.
Ditianasin.
Sick 13 years old boy complaint of a pain and edema of ankle joints, gripes and frequent urination,
reddening of a conjunctiva of eyes. What is your preliminary diagnosis?
*Reiter's disease.
Juvenile pseudorheumatism.
Infectious-allergic polyarthritis.
Systemic lupus erythematosus.
59